Sie sind auf Seite 1von 90

#1

G.R. No. 208170, August 20, 2014


PEOPLE OF THE PHILIPPINES, Plaintiff-Appellee, v. PETRUS YAU A.K.A.
“JOHN” AND “RICKY” AND SUSANA YAU Y SUMOGBA A.K.A. “SUSAN”,
Accused-Appellants

Topic: Article 18 RPC: Accomplice

FACTS:
Alastair Onglingswam, hailed a taxi cab and noted that while he was on the phone,
appellant Petrus Yau would from time to time looked and conversed on him as if he was
also being spoken to. He thereafter felt groggy and he woke up, his head was covered,
was handcuffed and chained. He was then informed that he was being kidnapped for
ransom and that he will be allowed to make phone calls to his family and friends. During
his 22 days of captivity, he was allowed to communicate with his family almost daily to
prove he was still alive and was served by meals almost 5x a day either by Joh or Susan
Yao. Subsequently thereafter, he was rescued by PACER (Police Anti-Crime and
Emergency Response Task Force) RTC convicted appellants guilty beyond reasonable
doubt. CA affirmed.

ISSUE: Whether Susana Yau is also considered as principal of the crime of kidnapping
for ransom

Ruling:
NO. Susan is liable only as an accomplice to the crime as correctly found by the lower
courts. It must be emphasized that there was no evidence indubitably proving that Susana
participated in the decision to commit the criminal act. The only evidence the prosecution
had against her was the testimony of Alastair to the effect that he remembered her as the
woman who gave food to him or who accompanied his kidnapper whenever he would
bring food to him every breakfast, lunch and dinner. in accordance with Article 18 of the
RPC, in order that a person may be considered an accomplice, namely, (1) that there be a
community of design; that is, knowing the criminal design of the principal by direct
participation, he concurs with the latter in his purpose; (2) that he cooperates in the
execution by previous or simultaneous act, with the intention of supplying material or
moral aid in the execution of the crime in an efficacious way; and (3) that there be a
relation between the acts done by the principal and those attributed to the person
charged as accomplice.

In the case at bench, Susana knew of the criminal design of her husband, Petrus, but she
kept quiet and never reported the incident to the police authorities. Instead, she stayed
with Petrus inside the house and gave food to the victim or accompanied her husband
when he brought food to the victim. Susana not only countenanced Petrus’ illegal act, but
also supplied him with material and moral aid. It has been held that being present and
giving moral support when a crime is being committed make a person responsible as an
accomplice in the crime committed. As keenly observed by the RTC, the act of giving
food by Susana to the victim was not essential and indispensable for the perpetration of
the crime of kidnapping for ransom but merely an expression of sympathy or feeling of
support to her husband. Moreover, this Court is guided by the ruling in People v. De Vera,
where it was stressed that in case of doubt, the participation of the offender will be
considered as that of an accomplice rather than that of a principal.

https://lookaside.fbsbx.com/file/Book%20I_Consolidated%20C…hqsTJHSGnedWZ0OSOpxZmrtUVtu6RtpxVMMpL7Kt2IHuRsCvg 22/10/2018, 9W25 PM


Page 1 of 90
https://lookaside.fbsbx.com/file/Book%20I_Consolidated%20C…hqsTJHSGnedWZ0OSOpxZmrtUVtu6RtpxVMMpL7Kt2IHuRsCvg 22/10/2018, 9W25 PM
Page 2 of 90
#2
People vs. Gonzales
G.R. No. 80762, March 19, 1990

Facts: Custodio Gonzales, Sr., his wife Fausta Gonzales, and four (4) others were
convicted of murder under Art. 248 of the RPC. Autopsy report showed that the victim,
Lloyd Peñacerrada, 44, landowner, sustained 16 wounds, five of which were fatal. Except
for Gonzales, Sr., 65 years old, and the one who still remained at large, all the others
withdrew their appeal and instead pursued their application for parole. Conviction was on
the basis of an account of one Jose Huntoria, the self-proclaimed eyewitness. On cross-
examination, however, Huntoria admitted that he could not determine who among the the
accused did the stabbing and/or hacking and what particular weapon was used by each of
them.
Issue: Whether criminal liability was incurred by appellant Gonzales, Sr. to sustain his
conviction.

Held: No. One of the means by which criminal liability is incurred is through the
commission of a felony under Article 3 of the RPC. The elements of felonies in general
are: (1) there must be an act or omission; (2) the act or omission must be punishable
under the Revised Penal Code; and (3) the act is performed or the omission incurred by
means of deceit or fault. There is nothing in the findings which would categorize the
criminal liability of appellant as principal by direct participation under Art. 17, par. 1 of
the RPC. Likewise, there is nothing in the evidence that inculpates him by inducement
under par. 2 of the same Art. 17, or by indispensable cooperation under par. 3 thereof.
The guilt of the accused has not been proven beyond reasonable doubt thus he was
acquitted.

https://lookaside.fbsbx.com/file/Book%20I_Consolidated%20C…hqsTJHSGnedWZ0OSOpxZmrtUVtu6RtpxVMMpL7Kt2IHuRsCvg 22/10/2018, 9W25 PM


Page 3 of 90
#3
People of the Philippines vs. Isabelo Puno and Enrique Amurao
G.R. No. 97471
February 17, 1993

FACTS:
Mrs. Maria Socorro Mutuc-Sarmiento owned a bakeshop in Araneta Avenue,
Quezon city. One day in January, Isabelo Puno, the personal driver of Sarmiento’s
husband, said that he would be substituting for Sarmiento’s due to an emergency.
On the way home, Puno went off route and stopped. That’s when Enrique
Amurao, Puno’s nephew, entered the car and pointed a gun toward Mutuc-Sarmiento.
Puno and Amurao demanded money from Mutuc-Sarmiento who gave them 7,000 pesos.
The two demanded more money, to which the victim obliged by issuing three checks in
favor of the two.
When they were supposed to go back to Valle Verde as they have agreed, Puno
suddenly went off course. This prompted Sarmiento to jump out of the car and hail a van
to escape. The accused, however, claims that Sarmiento did not jump out. They claim
that they stopped north bound, allowed the victim to step out, and waited for her to get a
ride home. The injury she obtained was caused when she fell down and stubbed her toe
while crossing the highway.
Pun and Amurao were arrested and were charged with kidnapping for ransom.
They pleaded not guilty. The lower court ruled that they were guilty of robbery with
extortion committed on a highway under Presidential Decree No. 532 and was sentenced
to jail to a term of reclusion perpetua.

ISSUE:
Whether or not there was intent to commit the crime of kidnapping on the part of
Puno and Amurao.

HELD:
No. There is no showing whatsoever that appellants had any motive, nurtured
prior to or at the time they committed the wrongful acts against complainant, other than
the extortion of money from her under the compulsion of threats or intimidation.
For kidnapping to exist, there must be indubitable proof that the actual intent of
the malefactors was to deprive the offended party of her liberty.
In the case, the restraint of her freedom of action was merely an incident in the
commission of another offense primarily intended by the offenders as shown in his
testimony when asked why he did not bring her to Valle Verde as they have agreed upon
when Mutuc-Sarmiento gave them the checks: “Because while we were on the way back,
it came to my mind that if we reach Balintawak or some other place along the way, we
might be apprehended by the police. So when we reached Santa Rita exit, I told her:
“Mam, we will already stop and allow you to get out of the car.”

#5
US vs Ah Chong

Facts:
The accused, Ah Chong, was employed as a cook in Fort Mckinley and was sharing the

https://lookaside.fbsbx.com/file/Book%20I_Consolidated%20C…hqsTJHSGnedWZ0OSOpxZmrtUVtu6RtpxVMMpL7Kt2IHuRsCvg 22/10/2018, 9W25 PM


Page 4 of 90
house with the deceased, Pascual Gualberto, who was employed as a house boy. The door
of the room they were occupying was not furnished with a permanent lock, and as a
measure of security, they fasten the door by propping a chair against it. One evening, Ah
Chong was suddenly awakened by someone trying to force open the door of their room.
The deceased and the accused had an understanding that when either returned late at
night, he should knock at the door and acquaint his companion with his identity. Ah
Chong sat up in bed and called out twice, “Who is there?” but heard no answer. The room
was quite dark, and as there had been recent robberies in Fort McKinley, fearing that the
intruder was a robber or a thief, he leaped to his feet and called out. “If you enter the
room, I will kill you.” Suddenly, he was struck by the edge of the chair which had been
placed against the door. Believing that he was being attacked, he seized a common
kitchen knife which he kept under his pillow and wildly struck and fatally wounded the
intruder who turned out to be his roommate, Pascual.

ISSUE:
Whether or not the accused was criminally liable.

HELD:
No. The rule is that one is not criminally liable if he acted without malice (criminal
intent), negligence, and imprudence. In the present case, the accused acted in good faith,
without malice or criminal intent, in the belief that he was doing no more than exercising
his legitimate right of self-defense. Had the facts been as he believed them to be, he
would have been wholly exempt from criminal liability on account of his act. Moreover,
the accused cannot be said to have been negligent or reckless as the facts as he saw them
threatens his person and his property. Under such circumstances, there is no criminal
liability, as the ignorance or mistake of fact was not due to negligence or bad faith.

#6
THE PEOPLE OF THE PHILIPPINES, plaintiff-appellee,
vs.
ROBERTO ESTRADA, accused-appellant.
G.R. No. 130487 (June 19, 2000)

FACTS:
In the morning of December 27, 1994, at the St. Johns Cathedral, Dagupan City,

https://lookaside.fbsbx.com/file/Book%20I_Consolidated%20C…hqsTJHSGnedWZ0OSOpxZmrtUVtu6RtpxVMMpL7Kt2IHuRsCvg 22/10/2018, 9W25 PM


Page 5 of 90
the sacrament of confirmation was being performed by the Roman Catholic Bishop of
Dagupan City. At 11:00 A.M., while the Bishop was giving his blessing, a man from the
crowd went up and walked towards the center of the altar. He stopped beside the Bishops
chair, turned around and, in full view of the Catholic faithful, sat on the Bishops chair.
Crisanto Santillan, who was assisting the Bishop at the rites, saw accused-appellant.
Santillan approached accused-appellant and requested him to vacate the Bishops chair.
Gripping the chairs armrest, accused-appellant replied in Pangasinese: No matter what
will happen, I will not move out! Hearing this, Santillan moved away.
Some of the churchgoers summoned Rogelio Mararac, the security guard at the
cathedral. Mararac went near accused-appellant and told him to vacate the Bishops chair.
Mararac grabbed his nightstick and used it to tap accused-appellants hand on the armrest.
Appellant did not budge. Again, Mararac tapped the latters hand. Still no reaction.
Mararac was about to strike again when suddenly accused-appellant drew a knife from
his back, lunged at Mararac and stabbed him, hitting him below his left throat. Mararac
fell. Mararac was brought to the hospital where he expired a few minutes upon arrival.

ISSUE
Whether or not the lower court erred in finding accused-appellant guilty of the
crime charged, despite clear and convincing evidence on record, supporting his plea of
insanity?

HELD
In the eyes of the law, insanity exists when there is a complete deprivation of
intelligence in committing the act. Mere abnormality of the mental faculties will not
exclude imputability. The accused must be so insane as to be incapable of entertaining a
criminal intent. He must be deprived of reason and act without the least discernment
because there is a complete absence of the power to discern or a total deprivation of
freedom of the will.
Since the presumption is always in favor of sanity, he who invokes insanity as an
exempting circumstance must prove it by clear and positive evidence. And the evidence
on this point must refer to the time preceding the act under prosecution or to the very
moment of its execution.
​In the case at bar, there is no direct proof that accused-appellant was afflicted with
insanity at the time he killed Mararac. The absence of direct proof, nevertheless, does not
entirely discount the probability that appellant was not of sound mind at that time.
Accused-appellants competence to stand trial must be properly ascertained to enable him
to participate in his trial meaningfully.

#9
People vs. Daniel Pinto Jr. ,204 SCRA 9, G.R. No. 39519 November 21, 1991

Facts:
​On December 25, 1970, the Legazpi City Police secured from the City Court of
Legazpi a warrant for the search of the house and premises of Francisco Bello in
Mariawa, Legazpi City on the ground that the police had probable cause to believe that
Bello illegally possessed a garand rifle, a thompson submachinegun and two automatic
pistols.2 The police had earlier undertaken a surveillance of Bello on the basis of
information it had received that he was conducting an “obstacle course” or training men
for combat since October, 1970.
​As an aftermath of the mission of the Legazpi City Police Department to serve on
Christmas day in 1970 a search warrant on Francisco Bello who was allegedly training a
private army, patrolmen Daniel Pinto, Jr. and Narciso Buenaflor, Jr. were found guilty
beyond reasonable doubt by the then Circuit Criminal Court in said city, of killing not
only Bello but also 9year-old Richard Tiongson and Rosalio Andes and seriously

https://lookaside.fbsbx.com/file/Book%20I_Consolidated%20C…hqsTJHSGnedWZ0OSOpxZmrtUVtu6RtpxVMMpL7Kt2IHuRsCvg 22/10/2018, 9W25 PM


Page 6 of 90
wounding Maria Theresa Tiongson.
Issue: Whether or not the attending circumstances warrants the conviction of the
accused.
Doctrines:
1. Requisites of justify-ing circumstance of fulfillment of a duty.—ln order that the
justifying circumstance of fulfillment of a duty under Article 11 of the Revised
Penal Code may be successfully invoked, the defense has to prove that these two
requisites are present: (a) the offender acted in the performance of a duty and (b)
the injury or offense committed be the necessary consequence of the due
performance or lawful exercise of such duty. In the absence the absence of the
second requisite, the justification becomes an incomplete one thereby converting
it into a mitigating circumstance under Articles 13 and 69 of the same Code.
Admittedly, the appellants and the rest of the police force involved, originally set
out to perform a legal duty: the service of a search warrant on Bello.

2. Mistake in identity of victims.—The fact that the victims were different from the
ones the appellants intended to injure cannot save them from conviction.
Aberratio ictus or mistake in the identity of the victim carries the same gravity as
when the accused zeroes in on his intended victim. The main reason behind this
conclusion is the fact that the accused had acted with such a disregard for the life
of the victim(s)—without checking carefully the latter’s identity—as to place
himself on the same legal plane as one who kills another willfully, unlawfully and
feloniously. Neither may the fact that the accused made a mistake in killing one
man instead of another be considered a mitigating circumstance.

3. Conspiracy.—It is not even necessary to pinpoint who between Pinto and Buenaflor
actually caused the death of Richard or the wounding of Maria Theresa in the
presence of proof beyond reasonable doubt that they acted in conspiracy with
each other. Prior agreement between the appellants to kill their intended victim is
not essential to prove conspiracy as the same may be inferred from their own acts
showing joint purpose and design. In this case, such unity of purpose and design
is shown by the fact that only the two of them fired their guns when the Anduiza
jeep with the Tiongsons passed by. This they did in defiance of the order of their
superior not to shoot unless ordered to do so. Conspiracy having been proved, the
guilt or culpability is imposable on both appellants in equal degrees.

4. Elements of Self-defense.—Under Article 11 (1) of the Rules of Court, an accused


must prove the presence of all the following elements of said exempting
circumstance: (a) unlawful aggression, (b) reasonable necessity of the means
employed to prevent or repel it, and (c) lack of sufficient provocation on the part
of the person defending himself. The presence of unlawful aggression is a
condition sine qua non. There can be no self-defense, complete or incomplete,
unless the victim has committed an unlawful aggression against the person
defending himself.

Qualifying circumstance of treachery.—We agree with the trial court that treachery
attended the commission of all four crimes in this case. The killing of Richard Tiongson,
Francisco Bello and Rosalio Andes as well as the wounding of Maria Theresa Tiongson
were all so sudden that all of them were left defenseless. This is shown not only by the
testimonial evidence of the commission of the crimes but also by the nature and location
of the wounds of all the victims. The presence of treachery qualifies the killings to

https://lookaside.fbsbx.com/file/Book%20I_Consolidated%20C…hqsTJHSGnedWZ0OSOpxZmrtUVtu6RtpxVMMpL7Kt2IHuRsCvg 22/10/2018, 9W25 PM


Page 7 of 90
murder and the wounding of Maria Theresa to frustrated murder. Nighttime, however,
may not be appreciated as there is no proof that it was specifically sought in the
commission of the crime and therefore we deem it absorbed by treachery.

#10
People of the Philippines v. Oanis and Galanta
G.R. No. L-47722, July 27, 1943

Facts:

While Tecson was sleeping in his room with his back towards the door, Oanis and
Galanta, on sight, fired at him simultaneously or successively, believing him to be
Anselmo Balagtas but without having made previously any reasonable inquiry as to his
identity. Awakened by the gunshots, Irene saw her paramour already wounded, and
looking at the door where the shots came, she saw the defendants still firing at him.
Shocked by the entire scene. Irene fainted; it turned out later that the person shot and
killed was not the notorious criminal Anselmo Balagtas but a peaceful and innocent
citizen named Serapio Tecson, Irene's paramour.

Issues:

Whether the defense of mistake of fact is tenable?

Ruling/Ratio:

No. The maxim is ignorantia facti excusat, but this applies only when the mistake is
committed without fault or carelessness. In support of the theory of non-liability by

https://lookaside.fbsbx.com/file/Book%20I_Consolidated%20C…hqsTJHSGnedWZ0OSOpxZmrtUVtu6RtpxVMMpL7Kt2IHuRsCvg 22/10/2018, 9W25 PM


Page 8 of 90
reasons of honest mistake of fact, appellants rely on the case of U.S. v. Ah Chong, 15
Phil., 488. The maxim is ignorantia facti excusat, but this applies only when the mistake
is committed without fault or carelessness. In the Ah Chong case, defendant therein after
having gone to bed was awakened by someone trying to open the door. He called out
twice, "who is there," but received no answer. Fearing that the intruder was a robber, he
leaped from his bed and called out again., "If you enter the room I will kill you." But at
that precise moment, he was struck by a chair which had been placed against the door and
believing that he was then being attacked, he seized a kitchen knife and struck and fatally
wounded the intruder who turned out to be his room-mate.

In the instant case, appellants, unlike the accused in the instances cited, found no
circumstances whatsoever which would press them to immediate action. The person in
the room being then asleep, appellants had ample time and opportunity to ascertain his
identity without hazard to themselves, and could even effect a bloodless arrest if any
reasonable effort to that end had been made, as the victim was unarmedThis, indeed, is
the only legitimate course of action for appellants to follow even if the victim was really
Balagtas, as they were instructed not to kill Balagtas at sight but to arrest him, and to get
him dead or alive only if resistance or aggression is offered by him.

#12
Edmundo Escamilla y Jugo vs. People of the
Philippines G.R. No. 188551, February 27, 2013

Topic: Physical Impossibility, Denial, Alibi

FACTS:

Petitioner has a house with a sari-sari store along Arellano Street, Manila. The victim,
Virgilio Mendol (Mendol), is a tricycle driver whose route traverses the road where
petitioner's store is located.

Around 2:00 a.m. of 01 August 1999, a brawl ensued at the comer of Estrada and
Arellano Streets, Manila. Mendol was about to ride his tricycle at this intersection while
facing Arellano Street. Petitioner, who was standing in front of his store, 30 meters away
from Mendol, shot the latter four times, hitting him once in the upper right portion of his
chest. The victim was brought to Ospital ng Makati for treatment and survived because of
timely medical attention.

Thereafter, an Information was filed charging petitioner with frustrated homicide. During
trial, the prosecution presented the testimonies of Mendol, Joseph Velasco (Velasco) and
Iluminado Garcelazo (Garcelazo), who all positively identified petitioner as the shooter
of Mendol. The doctor who attended to the victim also testified. The documentary
evidence presented included a sketch of the crime scene, the Medical Certificate issued
by the physician, and receipts of the medical expenses of Mendol when the latter was
treated for the gunshot wound.

The RTC found petitioner guilty and held that the positive testimonies of eyewitnesses
deserve far more weight and credence than the defense of alibi. Petitioner then filed a

https://lookaside.fbsbx.com/file/Book%20I_Consolidated%20C…hqsTJHSGnedWZ0OSOpxZmrtUVtu6RtpxVMMpL7Kt2IHuRsCvg 22/10/2018, 9W25 PM


Page 9 of 90
Notice of Appeal with the CA but the latter upheld the decision of the RTC.

ISSUE:
I. Whether the prosecution established petitioner’s guilt beyond reasonable doubt.

AI. Whether a defense of alibi, when corroborated by a disinterested party, overcomes


the positive identification by three witnesses.

HELD:
1. YES. Petitioner’s identity was proven with moral certainty. He was positively
identified by three witnesses as the shooter in the incident. According to the Court, a
categorical and consistently positive identification of the accused, without showing of ill
motive on the part of the eyewitnesses, prevails over denial. In the present case, none of
the witnesses had shown any ulterior motive to testify against petitioner.

There was also intent to kill. The intent to kill was simultaneous with the infliction of
injuries. Using a gun, petitioner shot the victim in the chest. Despite a bloodied right
upper torso, the latter still managed to run towards his house to ask for help. Nonetheless,
petitioner continued to shoot at him three more times, albeit unsuccessfully.

2. NO. In order for alibi to prosper, petitioner must establish by clear and convincing
evidence that, first, he was in another place at the time of the offense; and, second, it was
physically impossible for him to be at the scene of the crime. Petitioner was unable to
establish that he was at home at the time of the offense. His wife did not even know if he
was at home when the shooting happened. Further, his home was just in front of the street
where the shooting occurred.

Physical impossibility refers to the distance between the place where the accused was
when the crime transpired and the place where it was committed, as well as the facility of
access between the two places. Petitioner failed to prove the physical impossibility of his
being at the scene of the crime at the time in question.

Petitioner proffers the alibi that he was at home, instead of showing the impossibility of
his authorship of the crime. His alibi actually bolsters the prosecution's claim that he was
the shooter, because it placed him just a few steps away from the scene of the crime. The
charge is further bolstered by the testimony of his wife, who could not say with certainty
that he was at home at 2:00a.m.- the approximate time when the victim was shot.

https://lookaside.fbsbx.com/file/Book%20I_Consolidated%20C…hqsTJHSGnedWZ0OSOpxZmrtUVtu6RtpxVMMpL7Kt2IHuRsCvg 22/10/2018, 9W25 PM


Page 10 of 90
#13
[G.R. No. 129433. March 30, 2000]

PEOPLE OF THE PHILIPPINES, plaintiff,


vs.
PRIMO CAMPUHAN Y BELLO, accused.

Facts:

Campuhan was a helper in the business of the family of the victim, a 4-year-old
girl. One time, the mother of the victim heard the latter cry, “Ayoko!”, prompting
her to rush upstairs. There, she saw Campuhan kneeling before the victim, whose
pajamas and pany were already removed, while his short pants were down to his
knees. Campuhan was apprehended. Physical examination of the victim yielded
negative results. No evident sign of extra-genital physical injury was noted. Her
hymen was intact and its orifice was only .5 cm in diameter.

Trial court found him guilty of statutory rape and sentenced him to death.

Issue:

Whether or not Campuhan is guilty of statutory rape.

Held: NO.

The gravamen of the offense of statutory rape is carnal knowledge of woman


below 12 as provided in RPC 335(3). The victim was only 4 years old when the

https://lookaside.fbsbx.com/file/Book%20I_Consolidated%20C…hqsTJHSGnedWZ0OSOpxZmrtUVtu6RtpxVMMpL7Kt2IHuRsCvg 22/10/2018, 9W25 PM


Page 11 of 90
molestation took place, thus raising the penalty from “reclusion perpetua to
death” to the single indivisible penalty of death under RA 7659 Sec. 11, the
offended party being below 7 years old. In concluding that carnal knowledge took
place, full penetration of the vaginal orifice is not an essential ingredient, nor is
the rupture of hymen necessary; the mere touching of external genitalia by the
penis capable of consummating the sexual act is sufficient to constitute carnal
knowledge. But the act of touching should be understood as inherently part of the
entry of penis into the labias of the female organ, and not mere touching alone of
the mons pubis or the pudendum (the part instantly visible within the surface).

Absent any showing of the slightest penetration of the female organ, i.e.,
touching of either labia by the penis, there can be no consummated rape; at most,
it can only be attempted rape, if not acts of lasciviousness.

Here, the prosecution failed to discharge its onus of proving that Campuhan’s
penis was able to penetrate the victim’s vagina however slight. Also, there were
no external signs of physical injuries on the victim’s body to conclude that
penetration had taken place.

Issue #2:

What crime did Campuhan commit?

Held #2: ATTEMPTED RAPE.

Under RPC 6 in relation to RPC 335, rape is attempted when the offender
commences the commission of rape directly by overt acts, and does not perform
all acts of execution which should produce the crime of rape by reason of some
cause or accident other than his own spontaneous desistance. All the elements of
attempted rape are present in this case.

The penalty of attempted rape is 2 degrees lower than the imposable penalty of
death for the crime of statutory rape of minor below 7 years. Two degrees lower
is reclusion temporal, which is 12 years 1 day to 20 years.

Applying ISLAW, and in the absence of aggravating or mitigating circumstance,


the maximum penalty shall be medium period of reclusion temporal (14 years 8
months 1 day to 17 years 4 months), while the minimum is the penalty next lower
in degree – prision mayor (6 years 1 day to 12 years).

Issue #3:

May there be a crime of frustrated rape?

Held #3: NO.

In People vs Orita, SC finally did away with frustrated rape. Rape was
consummated from the moment the offender had carnal knowledge of the victim.
All elements of the offense were already present and nothing more was left for
the offender to do. Perfect penetration was not essential; any penetration of the

https://lookaside.fbsbx.com/file/Book%20I_Consolidated%20C…hqsTJHSGnedWZ0OSOpxZmrtUVtu6RtpxVMMpL7Kt2IHuRsCvg 22/10/2018, 9W25 PM


Page 12 of 90
female organ by the male organ, however slight, was sufficient.

For attempted rape, there was no penetration of the female organ because not all acts of
execution were performed or the offender merely commenced the commission of the
felony directly by overt acts.

#15
People vs Amadeo Peralta, et al.
G.R. No. L-19069
October 29, 1968

Facts:

On February 16, 1958, in the municipality of Muntinglupa, province of Rizal, two known
warring gangs inside the New Bilibid Prison as “Sigue-Sigue” and “OXO” were
preparing to attend a mass at 7 a.m. However, a fight between the two rival gangs caused
a big commotion in the plaza where the prisoners were currently assembled. The fight
was quelled and those involved where led away to the investigation while the rest of the
prisoners were ordered to return to their respective quarters.

In the investigation, it was found out that the accused, “OXO” members, Amadeo Peralta,
Andres Factora, Leonardo Dosal, Angel Paramog, Gervasio Larita and Florencio Luna
(six among the twenty-two defendants charged therein with multiple murder), are also
convicts confined in the said prisons by virtue of final judgments.

They conspired, confederated and mutually helped and aided each other, with evident
premeditation and treachery, all armed with deadly weapons, did, then and there,
willfully, unlawfully and feloniously killed “Sigue-Sigue” sympathizers Jose Carriego,
Eugenio Barbosa and Santos Cruz, also convicts confined in the same institution, by
hitting, stabbing, and striking them with ice picks, clubs and other improvised weapons,
pointed and/or sharpened, thereby inflicting upon the victims multiple serious injuries
which directly caused their deaths.

Issues

(a) Whether of not conspiracy attended the commission of the multiple murder?

(b) Whether or not an aggravating circumstance of quasi-recidivism is present in the


commission of the crime?

Held:

https://lookaside.fbsbx.com/file/Book%20I_Consolidated%20C…hqsTJHSGnedWZ0OSOpxZmrtUVtu6RtpxVMMpL7Kt2IHuRsCvg 22/10/2018, 9W25 PM


Page 13 of 90
A conspiracy exists when two or more persons come to an agreement concerning the
commission of a felony and decide to commit it. Generally, conspiracy is not a crime
unless when the law specifically provides a penalty thereof as in treason, rebellion and
sedition. However, when in resolute execution of a common scheme, a felony is
committed by two or more malefactors, the existence of a conspiracy assumes a pivotal
importance in the determination of the liability of the perpetrators. Once an express or
implied conspiracy is proved, all of the conspirators are liable as co-principals regardless
of the extent and character of their respective active participation in the commission of
the crime/s perpetrated in furtherance of the conspiracy because in contemplation of law
the act of one is the act of all.

The collective criminal liability emanates from the ensnaring nature of conspiracy. The
concerted action of the conspirators in consummating their common purpose is a patent
display of their evil partnership, and for the consequences of such criminal enterprise
they must be held solidarity liable. However, in order to hold an accused guilty as co-
principal by reason of conspiracy, it must be established that he performed an overt act in
furtherance of the conspiracy, either by actively participating in the actual commission of
the crime, or by lending moral assistance to his co-conspirators by being present at the
scene of the crime, or by exerting moral ascendancy over the rest of the conspirators as to
move them to executing the conspiracy.

Conspiracy alone, without execution of its purpose, is not a crime punishable by law,
except in special instances (Article 8, Revised Penal Code) which, do not include
robbery.

Reverting now to the case at bar, the trial court correctly ruled that conspiracy attended
the commission of the murders. To wit, although there is no direct evidence of
conspiracy, the court can safely say that there are several circumstances to show that the
crime committed by the accused was planned. First, all the deceased were Tagalogs and
members of sympathizers of “Sigue-Sigue” gang (OXO members were from either
Visayas or Mindanao), singled out and killed thereby, showing that their killing has been
planned. Second, the accused were all armed with improvised weapons showing that they
really prepared for the occasion. Third, the accused accomplished the killing with team
work precision going from one brigade to another and attacking the same men whom they
have previously marked for liquidation and lastly, almost the same people took part in the
killing of the Carriego, Barbosa and Cruz.

In view of the attendance of the special aggravating circumstances of quasi-recidivism, as


all of the six accused at the time of the commission of the offenses were serving
sentences in the New Bilibid Prison by virtue of convictions by final judgments that
penalty for each offense must be imposed in its maximum period, which is the mandate
of the first paragraph of article 160 of the RPC. Hence, severe penalty imposed on a
quasi-recidivist is justified because of the perversity and incorrigibility of the crime.

Accordingly, the judgment a quo is hereby modified as follows: Amadeo Peralta, Andres
Factora, Leonardo Dosal, Angel Paramog, Gervasio Larita and Florencio Luna are each
pronounced guilty of three separate and distinct crimes of murder, and are each sentenced
to three death penalties; all of them shall, jointly and severally, indemnify the heirs of
each of the three deceased victims in the sum of P12,000; each will pay one-sixth of the
costs.

https://lookaside.fbsbx.com/file/Book%20I_Consolidated%20C…hqsTJHSGnedWZ0OSOpxZmrtUVtu6RtpxVMMpL7Kt2IHuRsCvg 22/10/2018, 9W25 PM


Page 14 of 90
#16
PEOPLE OF THE PHILIPPINES vs. EDWIN DE VERA y GARCIA, et. al.
G.R. No. 128966 August 18, 1999
FACTS:
Cacao saw a car passing by driven by Capulong with four passengers. Among them were
Florendo and Garcia. Cacao then heard unintelligible voices then saw Capulong being
dragged by Florendo to a grassy place while Florendo was holding a gun. After firing the
gun and hitting between Capulong’s eyes, Florendo and companions fled to different
directions. De Vera admitted that he was with Florendo’s group but he had no part in the
killing and merely accompanied to Filinvest the other accused and Florendo.
ISSUE: Whether De Vera who acted as a lookout should be treated as accomplice. –
YES.
RULING:
De Vera’s presence was no innocuous knowing that Florendo intended to kill the victim
and that the three co-accused were carrying weapons, he acted as a lookout to watch for
passerby. He was not an innocent spectator; he was at the locus criminis in order to aid
and abet the commission of the crime. These facts did not, however, make him a
conspirator at most he was only an accomplice.
The Revised Penal Code defines an accomplice as those persons who, not being included
in Art. 17, cooperate in the execution of the offense by previous or simultaneous acts.
The court has held that an accomplice is one who knows the criminal design of the
principal and cooperate knowingly or intentionally therewith by an act which even if not
rendered, the crime would be committed just the same. To hold a person liable as an
accomplice, two elements must be present: (1) the “community” of the criminal design
that is knowing the criminal design of the principal by direct participation, he concurs
with the latter in his purpose and (2) the performance of previous simultaneous acts that
are not indispensable to the commission of the crime.
Conspirators and accomplices have one thing in common: they know and agree with the
criminal design. Conspirators, however, know the criminal intention because they
themselves have decided upon such course of action. Accomplices come to know about it
only after the principals have reached the decision and only then do they agree to
cooperate in its execution. Conspirators decide that the crime should be committed;
accomplices merely concur in it. They merely assent to the plan and cooperate in its
accomplishment. Conspirators are authors of a crime; accomplices are merely their
instruments who perform acts not essential to the perpetration of the offense.

#17
People vs Labiaga
GR 202867
July 15,2013

I. Facts: Gregorio and his 2 daughters, Judy ang Glenelyn, were in their home.
Gregorio stepped outside while Glenelyn was in their store which was part of
their house. Shortly thereafter, appellant shot Gregorio who called Judy for
help. When Judy and Glenylun rushed to Gregorio's aid, appellant shot Judy

https://lookaside.fbsbx.com/file/Book%20I_Consolidated%20C…hqsTJHSGnedWZ0OSOpxZmrtUVtu6RtpxVMMpL7Kt2IHuRsCvg 22/10/2018, 9W25 PM


Page 15 of 90
in the abdomen. The two accused were standing behind the appellant.
Appellant said that Judy was already dead when the three fled the crime scene.

II. Whether or not the acts of the appellant towards Gregorio constitute frustrated
murder.

III. NO. In a frustrated felony,the offender has performed all the acts of execution
which should produce the felony as a consequence; whereas in attempted
felony,the offender merely commences the commission of a felony directly by
overt acra and does not perform all the acts of execution. Also,in frustrates
felony,the reason for the non-accomplishment of the crime is some cause
independent of the will of the perpetrator; on the other hand,in attempted
felony, the reason for the non-fulfillment of the crime is a cause or accident
other than the offender's own spontaneous desistance. In frustrates
murder,there must be evidence showing that the wound would have been fatal
were it not for timely medical intervention. In the instant case, it does not
appear that the wound sustained by Gregorio Conde was mortal. This was
admitted by Dr. Edwin Figura, who examined Gregorio after the shooting
incident.

#17.a
Gloria Macapagal-Arroyo vs.People of the Philippines and the Sandiganbayan, G. R.
No. 220598, 19 July 2016
Bersamin, J:

FACTS:

https://lookaside.fbsbx.com/file/Book%20I_Consolidated%20C…hqsTJHSGnedWZ0OSOpxZmrtUVtu6RtpxVMMpL7Kt2IHuRsCvg 22/10/2018, 9W25 PM


Page 16 of 90
The Court resolves the consolidated petitions for certiorari separately filed by former
President Gloria Macapagal-Arroyo and Philippine Charity Sweepstakes Office (PCSO)
Budget and Accounts Manager Benigno B. Aguas.
On July 10, 2012, the Ombudsman charged in the Sandiganbayan former President Gloria
Macapagal-Arroyo (GMA) and PCSO Budget and Accounts Manager Aguas (and some
other officials of PCSO and Commission on Audit whose charges were later dismissed by
the Sandiganbayan for conspiracy to commit plunder, as defined by, and penalized under
Section 2 (b) of Republic Act (R.A.) No. 7080, as amended by R.A. No. 7659.
The information reads: That during the period from January 2008 to June 2010 or
sometime prior or subsequent thereto xxx accused Gloria Macapagal-Arroyo, the then
President of the Philippines xxx Benigno Aguas, then PCSO Budget and Accounts
Manager, all public officers committing the offense in relation to their respective offices
and taking undue advantage of their respective official positions, authority, relationships,
connections or influence, conniving, conspiring and confederating with one another,
did then and there willfully, unlawfully and criminally amass, accumulate and/or acquire,
directly or indirectly, ill-gotten wealth in the aggregate amount or total value of
PHP365,997,915.00, more or less, [by raiding the public treasury].
Thereafter, accused GMA and Aguas separately filed their respective petitions for
bail which were denied by the Sandiganbayan on the ground that the evidence of guilt
against them was strong.
After the Prosecution rested its case, accused GMA and Aguas then separately filed their
demurrers to evidence asserting that the Prosecution did not establish a case for plunder
against them. The same were denied by the Sandiganbayan, holding that there was
sufficient evidence to show that they had conspired to commit plunder. After the
respective motions for reconsideration filed by GMA and Aguas were likewise denied by
the Sandiganbayan, they filed their respective petitions for certiorari.

ISSUES:
Whether or not the State sufficiently established all the elements of the crime of plunder:
(a) Was there evidence of amassing, accumulating or acquiring ill-gotten wealth in the
total amount of not less than P50,000,000.00? (b) Was the predicate act of raiding the
public treasury alleged in the information proved by the Prosecution?

RULING:
No. No proof of amassing, or accumulating, or acquiring ill-gotten wealth of at least
Php50 Million was adduced against GMA and Aguas.
The corpus delicti of plunder is the amassment, accumulation or acquisition of ill-gotten
wealth valued at not less than Php50,000,000.00. The failure to establish the corpus
delicti should lead to the dismissal of the criminal prosecution.
As regards the element that the public officer must have amassed, accumulated or
acquired ill-gotten wealth worth at least P50,000,000.00, the Prosecution adduced no
evidence showing that either GMA or Aguas or even Uriarte, for that matter, had
amassed, accumulated or acquired ill-gotten wealth of any amount. There was also
no evidence, testimonial or otherwise, presented by the Prosecution showing even
the remotest possibility that the CIFs [Confidential/Intelligence Funds] of the PCSO
had been diverted to either GMA or Aguas, or Uriarte.
(b) The Prosecution failed to prove the predicate act of raiding the public treasury
(under Section 2 (b) of Republic Act (R.A.) No. 7080, as amended)
To discern the proper import of the phrase raids on the public treasury, the key is to look
at the accompanying words: misappropriation, conversion, misuse or malversation of
public funds [See Sec. 1(d) of RA 7080]. This process is conformable with the maxim of
statutory construction noscitur a sociis, by which the correct construction of a particular
word or phrase that is ambiguous in itself or is equally susceptible of various meanings
may be made by considering the company of the words in which the word or phrase is
found or with which it is associated. Verily, a word or phrase in a statute is always used in
association with other words or phrases, and its meaning may, therefore, be modified or
restricted by the latter. To convert connotes the act of using or disposing of another’s

https://lookaside.fbsbx.com/file/Book%20I_Consolidated%20C…hqsTJHSGnedWZ0OSOpxZmrtUVtu6RtpxVMMpL7Kt2IHuRsCvg 22/10/2018, 9W25 PM


Page 17 of 90
property as if it were one’s own; to misappropriate means to own, to take something for
one’s own benefit; misuse means “a good, substance, privilege, or right used improperly,
unforeseeably, or not as intended;” and malversation occurs when “any public officer
who, by reason of the duties of his office, is accountable for public funds or property,
shall appropriate the same or shall take or misappropriate or shall consent, through
abandonment or negligence, shall permit any other person to take such public funds, or
property, wholly or partially.” The common thread that binds all the four terms together is
that the public officer used the property taken. Considering that raids on the public
treasury is in the company of the four other terms that require the use of the property
taken, the phrase raids on the public treasury similarly requires such use of the property
taken. Accordingly, the Sandiganbayan gravely erred in contending that the mere
accumulation and gathering constituted the forbidden act of raids on the public
treasury. Pursuant to the maxim of noscitur a sociis, raids on the public
treasury requires the raider to use the property taken impliedly for his personal
benefit.
As a result, not only did the Prosecution fail to show where the money went but,
more importantly, that GMA and Aguas had personally benefited from the same.
Hence, the Prosecution did not prove the predicate act of raids on the public
treasury beyond reasonable doubt.
WHEREFORE, the Court GRANTS the petitions for certiorari; ANNULS and SETS
ASIDE the resolutions issued in Criminal Case No. SB-12-CRM-0174 by the
Sandiganbayan on April 6, 2015 and September 10, 2015; GRANTS the petitioners’
respective demurrers to evidence; DISMISSES Criminal Case No. SB-12-CRM-0174 as
to the petitioners GLORIA MACAPAGAL-ARROYO and BENIGNO AGUAS for
insufficiency of evidence; ORDERS the immediate release from detention of said
petitioners; and MAKES no pronouncements.

#17.b
G.R. No. 123979 December 03, 1998
PEOPLE OF THE PHILIPPINES, PLAINTIFF-APPELLEE, VS., ALIPIO
SANTIANO, JOSE SANDIGAN, ARMENIA PILLUETA AND JOSE VICENTE
(JOVY) CHANCO ACCUSED-APPELLANTS.

Facts: Accused-appellants Alipio Santiano, Jose Sandigan, Armenia Pillueta and Jose
Vicente (Jovy) Chanco were indicted for the kidnapping with murder of Ramon John Dy
Kow, Jr., a detention prisoner at the Naga City Jail, in an amended Information, filed with
the Regional Trial Court (RTC) of Pili, Branch 32, Camarines Sur. That on or about the
27th day of December 1993 between 6:00 oclock to 7:00 oclock in the evening at
Barangay Palestina, Municipality of Pili, Province of Camarines Sur, Philippines and
within the jurisdiction of this Honorable Court, the above-named accused conspiring,
confederating and mutually helping one another with intent to kill, with treachery,
superior strength and evident premeditation, did then and there, willfully, unlawfully and
feloniously abduct, kidnap and bring into a secluded place at Palestina, Pili Camarines
Sur, one RAMON JOHN DY KOW, JR. and while there at attack and shoot with firearm
the said Ramon John Dy Kow, Jr. for several times hitting him on the different parts of
his body causing his instantaneous death.

Issue: When a complex crime has been charged in an information and the evidence fails
to support the charge on one of the component offenses, can the defendant still be
separately convicted of the other offense?

https://lookaside.fbsbx.com/file/Book%20I_Consolidated%20C…hqsTJHSGnedWZ0OSOpxZmrtUVtu6RtpxVMMpL7Kt2IHuRsCvg 22/10/2018, 9W25 PM


Page 18 of 90
Held: The question has long been answered in the affirmative. In United States vs.
Lahoylahoy and Madanlog, the Court has ruled to be legally feasible the conviction of an
accused on one of the offenses included in a complex crime charged, when properly
established, despite the failure of evidence to hold the accused of the other charge.
The fact alone that appellant Pillueta is an organic member of the NARCOM and
appellant Sandigan a regular member of the PNP would not exempt them from the
criminal liability for kidnapping. It is quite clear that in abducting and taking away the
victim, appellants did so neither in furtherance of official function nor in the pursuit of
authority vested in them. It is not, in fine, in relation to their office, but in purely private
capacity, that they have acted in concert with their co-appellants Santiano and Chanco.
The crime of kidnapping cannot be here absorbed by the charge of murder since the
detention of the victim is not shown to have been for the purpose of liquidating him.
Appellants themselves, in fact, all deny having killed the victim. And while the evidence
may have thus been found to be wanting by the trial court so as to equally hold appellants
responsible for the death of the victim, the Court is convinced that the court a quo did not
err in making them account for kidnapping. The circumstances heretofore recited indicate
the attendance of conspiracy among the appellants thereby making them each liable for
the offense.

#18
G.R. Nos. L-33466-67 April 20, 1983 PEOPLE OF THE PHILIPPINES, plaintiff-
appellee, vs.MAMERTO NARVAEZ, defendant-appellant.

Facts:
Mamerto Narvaez has been convicted of murder (qualified by treachery) of David
Fleischer and Flaviano Rubia. On August 22, 1968, Narvaez shot Fleischer and Rubia
during the time the two were constructing a fence that would prevent Narvaez from
getting into his house and rice mill. The defendant was taking a nap when he heard
sounds of construction and found fence being made. He addressed the group and asked
them to stop destroying his house and asking if they could talk things over. Fleischer
responded with "No, gadamit, proceed, go ahead." Defendant lost his "equilibrium," and
shot Fleisher with his shotgun. He also shot Rubia who was running towards the jeep
where the deceased's gun was placed. Prior to the shooting, Fleischer and Co. (the
company of Fleischer's family) was involved in a legal battle with the defendant and
other land settlers of Cotabato over certain pieces of property. At the time of the shooting,
the civil case was still pending for annulment (settlers wanted granting of property to
Fleisher and Co. to be annulled). At time of the shooting, defendant had leased his
property from Fleisher (though case pending and ownership uncertain) to avoid trouble.
On June 25, defendant received letter terminating contract because he allegedly didn't pay
rent. He was given 6 months to remove his house from the land. Shooting was barely 2
months after letter. Defendant claims he killed in defense of his person and property. CFI
ruled that Narvaez was guilty. Aggravating circumstances of evident premeditation offset
by the mitigating circumstance of voluntary surrender. For both murders, CFI sentenced
him to reclusion perpetua, to indemnify the heirs, and to pay for moral damages

ISSUES:
1. WON CFI erred in convicting defendant-appellant despite the fact that he acted in
defense of his person.
2. WON the court erred in convicting defendant-appellant although he acted in defence of

https://lookaside.fbsbx.com/file/Book%20I_Consolidated%20C…hqsTJHSGnedWZ0OSOpxZmrtUVtu6RtpxVMMpL7Kt2IHuRsCvg 22/10/2018, 9W25 PM


Page 19 of 90
his rights.

Held:
1.The courts concurred that the fencing and chiselling of the walls of the house of the
defendant was indeed a form of aggression on the part of the victim. However, this
aggression was not done on the person of the victim but rather on his rights to property.
On the first issue,the courts did not err. However, in consideration of the violation of
property rights, the courts referred to Art. 30 of the civil code recognizing the right of
owners to close and fence their land. Although is not in dispute, the victim was not in the
position to subscribe to the article because his ownership of the land being awarded by
the government was still pending, therefore putting ownership into question. It is
accepted that the victim was the original aggressor.
2. Yes. However, the argument of the justifying circumstance of self-defense is applicable
only if the 3 requirements are fulfilled. Art. 11(1) RPC enumerates these requisites: ·
Unlawful aggression. In the case at bar, there was unlawful aggression towards
appellant's property rights. Fleisher had given Narvaez 6 months and he should have left
him in peace before time was up, instead of chiseling Narvaez's house and putting up
fence. Art. 536 of the Civil Code also provides that possession may not be acquired
through force or intimidation; while Art. 539 provides that every possessor has the right
to be respected in his possession · Reasonable necessity of means employed to prevent or
repel attack. In the case, killing was disproportionate to the attack. · Lack of sufficient
provocation on part of person defending himself. Here, there was no provocation at all
since he was asleep Since not all requisites present, defendant is credited with the special
mitigating circumstance of incomplete defense, pursuant to Art. 13(6)RPC. These
mitigating circumstances are: voluntary surrender and passion and obfuscation (read p.
405 explanation) Crime is homicide (2 counts) not murder because treachery is not
applicable on account of provocation by the deceased. Also, assault was not deliberately
chosen with view to kill since slayer acted instantaneously. There was also no direct
evidence of planning or preparation to kill. Art. 249 RPC: Penalty for homicide is
reclusion temporal. However, due to mitigating circumstances and incomplete defense, it
can be lowered three degrees (Art. 64) to arresto mayor.

https://lookaside.fbsbx.com/file/Book%20I_Consolidated%20C…hqsTJHSGnedWZ0OSOpxZmrtUVtu6RtpxVMMpL7Kt2IHuRsCvg 22/10/2018, 9W25 PM


Page 20 of 90
#19
Flores vs. P, G.R. No. 181354, February 27, 2013

FACTS : On August 15, 1989, on the eve of the barangay fiesta in San Roque, Alaminos,
Laguna, certain visitors, Ronnie de Mesa, Noli de Mesa, Marvin Avenido, and Duran,
were drinking at the terrace of the house of Jesus. They started drinking at 8:30 o’clock in
the evening. Jesus, however, joined his visitors only at around 11:00 o’clock after he and
his wife arrived from Sta. Rosa, Laguna, where they tried to settle a problem regarding a
vehicular accident involving one of their children. The drinking at the terrace was
ongoing when Flores arrived with an M-16 armalite rifle.

Duran testified that Jesus stood up from his seat and met Flores who was heading towards
the terrace. After glancing at the two, who began talking to each other near the terrace,
Duran focused his attention back to the table. Suddenly, he heard several gunshots
prompting him to duck under the table. Right after the shooting, he looked around and
saw the bloodied body of Jesus lying on the ground. By then, Flores was no longer in
sight.

Flores' version is that he proceeded to the terrace of the house of Jesus, who was having a
drinking spree with four others. In a calm and courteous manner, Flores asked Jesus and
his guests to cease firing their guns as it was already late at night and to save their shots
for the following day’s fiesta procession. Flores claimed that despite his polite,
unprovocative request and the fact that he was a relative of Jesus and the barangay
chairman, a person in authority performing a regular routine duty, he was met with
hostility by Jesus and his guests. Jesus, who appeared drunk, immediately stood up and
approached him as he was standing near the entrance of the terrace. Jesus abruptly drew
his magnum pistol and poked it directly at his chest and then fired it. By a twist of fate,
he was able to partially parry Jesus’ right hand, which was holding the pistol, and was hit
on his upper right shoulder.

ISSUE: Whether or not the claim of self defense is justified.

https://lookaside.fbsbx.com/file/Book%20I_Consolidated%20C…hqsTJHSGnedWZ0OSOpxZmrtUVtu6RtpxVMMpL7Kt2IHuRsCvg 22/10/2018, 9W25 PM


Page 21 of 90
RULING: No. If the accused, however, admits killing the victim, but pleads self-defense,
the burden of evidence is shifted to him to prove such defense by clear, satisfactory and
convincing evidence that excludes any vestige of criminal aggression on his part. To
escape liability, it now becomes incumbent upon the accused to prove by clear and
convincing evidence all the elements of that justifying circumstance.

In this case, Flores does not dispute that he perpetrated the killing of Jesus by shooting
him with an M16 armalite rifle. To justify his shooting of Jesus, he invoked self-defense.
By interposing self-defense, Flores, in effect, admits the authorship of the crime. Thus, it
was incumbent upon him to prove that the killing was legally justified under the
circumstances.

To successfully claim self-defense, the accused must satisfactorily prove the concurrence
of the elements of self- defense. Under Article 11 of the Revised Penal Code, any person
who acts in defense of his person or rights does not incur any criminal liability provided
that the following circumstances concur: (1) unlawful aggression; (2) reasonable
necessity of the means employed to prevent or repel it; and (3) lack of sufficient
provocation on the part of the person defending himself.

The most important among all the elements is unlawful aggression. "There can be no self-
defense, whether complete or incomplete, unless the victim had committed unlawful
aggression against the person who resorted to self-defense." "Unlawful aggression is
defined as an actual physical assault, or at least a threat to inflict real imminent injury,
upon a person. In case of threat, it must be offensive and strong, positively showing the
wrongful intent to cause injury. It presupposes actual, sudden, unexpected or imminent
danger––not merely threatening and intimidating action. It is present only when the one
attacked faces real and immediate threat to one’s life." "Aggression, if not continuous,
does not constitute aggression warranting self-defense."

Indeed, the nature and number of the gunshot wounds inflicted upon Jesus further negate
the claim of self-defense by the accused. Records show that Jesus suffered four (4)
gunshot wounds in the different parts of his body. If there was any truth to Flores’ claim
that he merely acted in self-defense, his first shot on Jesus’ shoulder, which already
caused the latter to fall on the ground, would have been sufficient to repel the attack
allegedly initiated by the latter. But Flores continued shooting Jesus. Considering the
number of gunshot wounds sustained by the victim, the Court finds it difficult to believe
that Flores acted to defend himself to preserve his own life. "It has been held in this
regard that the location and presence of several wounds on the body of the victim provide
physical evidence that eloquently refutes allegations of self-defense."

"When unlawful aggression ceases, the defender no longer has any justification to kill or
wound the original aggressor. The assailant is no longer acting in self-defense but in
retaliation against the original aggressor." Retaliation is not the same as self-defense. In
retaliation, the aggression that was begun by the injured party already ceased when the
accused attacked him, while in self-defense the aggression still existed when the
aggressor was injured by the accused.

The means employed by a person claiming self-defense must be commensurate to the


nature and the extent of the attack sought to be averted, and must be rationally necessary
to prevent or repel an unlawful aggression. In this case, the continuous shooting by Flores
which caused the fatal gunshot wounds were not necessary and reasonable to prevent the
claimed unlawful aggression from Jesus as the latter was already lying flat on the ground

https://lookaside.fbsbx.com/file/Book%20I_Consolidated%20C…hqsTJHSGnedWZ0OSOpxZmrtUVtu6RtpxVMMpL7Kt2IHuRsCvg 22/10/2018, 9W25 PM


Page 22 of 90
after he was first shot on the shoulder.

#20
People v Gregorio
G.R. No. 109614

Facts:
May 7, 1986- Carlos Catorse together with his 15-year old son Romeo Catorse, arrived at
the two-storey house of appellant Adronico Gregorio, at Sitio Bug-as, Brgy. Sta. Cruz in
Murcia, Negros Occidental, to attend the wake of the latter’s grandson. When Carlos and
his son arrived, there were already people attending the wake. Downstairs, Adronico
Gregorio, et. al. were conversing, while upstairs, “Tunggak” (son of Adronico), Ricardo
Gregorio (brother of Adronico), et. al. were playing “pusoy” (Russian poker).
Persons attending the wake were requested by appellant Adronico to deposit with him
any weapon in their possession for safekeeping to avoid any trouble. Complying
therewith, Carlos handed over his “samurai” while John Villarosa and Remolito Calalas,
surrendered their knives, to Adronico.

May 8, 1986- while playing the Russian poker, appellant Ricardo Gregorio in a very loud
voice, reprimanded “Tunggak” from peeping at the cards of other players, but the son of
Adronico, shouted also in a very loud voice and wanted the game be stopped. When his
father overheard it, he summoned his son and boxed him several times.

In order to pacify the father and son from further aggravation, Carlos Catorse intervened
and begged Adronico to stop hurting his son and not to put him into shame before the
crowd. When suddenly, co-appellant Ricardo stealthily stabbed Carlos from behind using
the latter’s own samurai and thereafter hacked and stabbed him several times more in
different parts of his body. Right after the deceased fell to the ground, Adronico for his
part, repeatedly hacked the victim with bolo.

Romeo Catorse, son of the deceased, terrified of what he saw and ran out of the house.
Later, when Romeo returned to the house of Adronico Gregorio, he was joined by his
sister and younger brothers, together they found their father lying prostrate and dead.
When the police authorities arrived to the scene of the crime, to investigate, the
appellants already fled to another Sitio, but authorities pursued and succeeded in
apprehending them.

Upon arraignment, both accused entered separate pleas of not guilty for murdering Carlos
Catorse. Hence, another criminal case was instituted against Adronico Gregorio for the
murder of Marcelo Lo.

https://lookaside.fbsbx.com/file/Book%20I_Consolidated%20C…hqsTJHSGnedWZ0OSOpxZmrtUVtu6RtpxVMMpL7Kt2IHuRsCvg 22/10/2018, 9W25 PM


Page 23 of 90
Issue: Whether or not appellants can invoke self-defense in their criminal liabilities.

Held: Appellant Adronico Gregorio interposed self-defense to exculpate himself from


criminal liability, however, the trial court skeptic of the said plea, rejected the same. The
futility of invoking self-defense is likewise revealed in the testimonies of accused
Ricardo Gregorio and co-appellant brother. Ricardo’s testimony affirmed that it was
Carlos Catorse who aggravated them initially, supported by Adronico’s claim, that the
deceased first attacked his son and brother. The court held that not only are the foregoing
declarations incredible and incredulous but are innately false and fatuous.

It is axiomatic that for self-defense to prosper, the following requisites must concur: (1)
there must be unlawful aggression by the victim; (2) that the means employed to prevent
or repel such aggression were reasonable; and (3) that there was luck of sufficient
provocation on the part of the person defending himself.

The trial court agree that such aggravating circumstance of treachery (alevosia) may be
appreciated against the appellants. Treachery exist when an offender commits any of the
crimes against persons, employing means, methods or forms in the execution thereof
which tend to directly and specially ensure its execution, without risks to himself arising
from the defense which the offended party might make.

https://lookaside.fbsbx.com/file/Book%20I_Consolidated%20C…hqsTJHSGnedWZ0OSOpxZmrtUVtu6RtpxVMMpL7Kt2IHuRsCvg 22/10/2018, 9W25 PM


Page 24 of 90
#21
G.R. No. L-56358 October 26, 1990
PEOPLE OF THE PHILIPPINES, plaintiff-appellee, vs. LUIS B. TORING
DIOSDADO BERDON and CARMELO B. BERDIN, accused-appellants.

Facts: In the evening of May 25, 1980, a benefit dance was held at sitio Naga, Babag II,
Lapu-lapu City for the last canvassing of votes for the candidates for princesses who
would reign at the sitio fiesta. As one of the candidates was the daughter of Samuel
Augusto, he and the members of his family attended the affair.
Also present were members of the kwaknit gang, a group which was noted for their bird-
like way of dancing and their propensity for drunkenness and provoking trouble. Its
president, called the "alas" king, was Luis Toring. The group was then outside the
dancing area which was ringed by benches.
At around 10:45 p.m., Samuel's daughter was proclaimed the winner in the contest. Beer
and softdrinks having been served the parents of the candidates by the officers of the
Naga Chapel Association which took charge of the affair, Samuel was tipsy when, after
his daughter's proclamation, he stepped out of the dancing area to answer the call of
nature.
At that moment, barangay tanod Felix Berdin saw Luis Toring, Carmelo Berdin and
Diosdado Berdon proceed to a dark area while whispering to each other. Diosdado
2
Berdon handed a knife to Luis Toring, who then approached Samuel from behind, held
Samuel's left hand with his left hand, and with his right hand, stabbed with the knife the
3
right side of Samuel's abdomen. Upon seeing Felix running towards them, Luis Toring
pulled out the knife and, together with Carmelo Berdin and Diosdado Berdon, ran
towards the dark. Felix tried to chase the three but he was not able to catch them. He
returned to where Samuel had slumped and helped others in taking Samuel to the
hospital. An information for murder was filed against Toring.
RTC – discrediting Toring's claim that the killing of Samuel was justified because it was
done in defense of a stranger pursuant to Article 11 (3) of the Revised Penal Code. The
lower court found that Toring was the "aggressor acting in retaliation or revenge by
reason of a running feud or long-standing grudge" between the kwaknit gang and the
group of Samuel, who, being the son of the barangay captain, was a "power to be
reckoned with." It mentioned the fact that a year before the incident in question, Toring
was shot by Edgar Augusto (Samuel's brother) and hence, in his desire to avenge himself,
Toring, "needed but a little excuse to do away with the object of his hatred. The lower
court, however, ruled out the existence of conspiracy among the three accused on the
ground that there was no proof on what they were whispering about when Felix saw
them. Accordingly, it held that the accused have individual or separate liabilities for the
killing of Samuel: Toring, as a principal, Diosdado Berdon as an accomplice by his act of
giving Toring the knife, and Carmelo Berdin as an accessory for concealing the weapon.
It considered treachery as the qualifying circumstance to the killing, found no proof as to
allegation of evident premeditation but appreciated nighttime as an aggravating
circumstance.
Toring seeks his exoneration by contending that his assault on Samuel was justified
because he acted in defense of his first cousin, Joel Escobia. Joel Escobia, whose chin

https://lookaside.fbsbx.com/file/Book%20I_Consolidated%20C…hqsTJHSGnedWZ0OSOpxZmrtUVtu6RtpxVMMpL7Kt2IHuRsCvg 22/10/2018, 9W25 PM


Page 25 of 90
was hit with the butt of Samuel's shotgun, is the first cousin of Toring their fathers being
brothers.

Issue: WON the justifying circumstance of defense of a relative must be taken into
consideration.

Held: No.
The appreciation of the justifying circumstance of defense of a relative hinges, in this
case, on the presence of unlawful aggression on the part of the victim.
The presence of unlawful aggression on the part of the victim and the lack of proof of
provocation on the part of Toring notwithstanding, full credence cannot be given, to
Toring's claim of defense of a relative. Toring himself admitted in court that in 1979, he
was shot with a .22 caliber revolver by Edgar Augusto, Samuel's brother. It cannot be
said, therefore, that in attacking Samuel, Toring was impelled by pure compassion or
beneficence or the lawful desire to avenge the immediate wrong inflicted on his cousin.
Rather, he was motivated by revenge, resentment or evil motive because of a "running
feud" between the Augusto and the Toring brothers. As the defense itself claims, after the
incident subject of the instant case occurred, Toring's brother, Arsenio, was shot on the
leg by Edgar Augusto. Indeed, vendetta appears to have driven both camps to commit
unlawful acts against each other. Hence, under the circumstances, to justify Toring's act
of assaulting Samuel Augusto would give free rein to lawlessness.
The lower court correctly considered the killing as murder in view of the presence of the
qualifying circumstance of treachery. The suddenness of the assault rendered Samuel
helpless even to use his shotgun. We also agree with the lower court that conspiracy and
evident premeditation were not proven beyond reasonable doubt. Moreover, nighttime
cannot be considered as an aggravating circumstance. There is no proof that it was
purposely sought to insure the commission of the crime or prevent its discovery.
However, Toring should be credited with the privileged mitigating circumstance of
incomplete defense of relative and the generic mitigating circumstance of voluntary
surrender.

#22
ANDAL VS. SANDIGANBAYAN, 60159,
November 6, 1989

https://lookaside.fbsbx.com/file/Book%20I_Consolidated%20C…hqsTJHSGnedWZ0OSOpxZmrtUVtu6RtpxVMMpL7Kt2IHuRsCvg 22/10/2018, 9W25 PM


Page 26 of 90
FACTS:

The petitioner, Fausto Andal, a member of the Batangas Integrated National Police, has
appealed to this Court the decision of the Sandiganbayan in Criminal Case which found
him guilty of the crime of Homicide. Petitioner, was then a corporal whose duty shift was
from 4:00 o'clock in the afternoon to 12:00 o'clock midnight, was on patrol aboard a
tricycle driven by Police Pfc. Casiano Quinio in the evening of 25 September 1980. At
about 7:00 o'clock that night, he went to the pier located at Sta. Clara, Batangas City, to
check on one of his men, Pfc. Maximo Macaraig, who was stationed there, because the
said Macaraig had failed to report to police headquarters for briefing.

Upon reaching the police checkpoint at the pier, and upon seeing Macaraig, petitioner
asked Macaraig why he did not pass by police headquarters for briefing before
proceeding to his post. Macaraig replied that he did not have to report to police
headquarters since he already had his orders. Sensing trouble, Quinio drove away his
tricycle. Macaraig, however, followed them and told the petitioner: "You report, supsup
ka." Petitioner kept his cool and did not say anything. But, Quinio went to Macaraig to
pacify him. Thereafter, petitioner and Quinio went back to the poblacion of Batangas
City.

At about 11:00 o'clock that night, petitioner and Quinio parked their vehicle in front of
the Philbanking Building at P. Burgos Street, Batangas City. Quinio alighted from the
tricycle and joined Pat. Andres Perez and Pat. Pedro Banaag who were seated on a bench.
The petitioner also alighted from the tricycle and stood at the sidewalk near the bench.
After a few minutes, Macaraig arrived and went straight to the petitioner. He was furious
this time and demanded why the petitioner had embarrassed him in front of so many
people. The petitioner denied the charge and called Quinio to clear up matters with
Macaraig. Quinio told Macaraig that the petitioner did not utter defamatory words against
him and asked him to forget the incident. Macaraig did not say anything. But, he returned
to the petitioner and challenged him. Quinio again tried to pacify Macaraig and brought
him across the street. Still, Macaraig refused to be pacified and went to the petitioner
with a drawn gun in his hand.

Pointing the gun menacingly at the petitioner, Macaraig said: "Bumunot ka bumunot ka."
Petitioner, however, refused to fight, saying: "I cannot fight you because we are both
policemen" Macaraig, nevertheless, fired his gun pointblank at the petitioner, hitting the
latter in the middle aspect, lower right knee. Petitioner then lunged at Macaraig and they
grappled for possession of the gun. Petitioner was able to wrest the gun from Macaraig.
Thereafter, two (2) successive shots were fired and Macaraig fell to the ground. He was
brought to the hospital but he was dead on arrival.

ISSUE:

Whether Andal can validly invoke the self-defense.

RULING:

NO. The initial illegal aggression staged by deceased had ceased after he was disarmed
by accused. By then, accused a taller and bigger man than deceased had the upper hand.
He was in possession of the gun of deceased while the latter was unarmed. In fact, it was
probably because of this circumstance that deceased moved backward. Aside from
accused, his son who dared to fight deceased was there, not to say Pat. Perez and Quinio

https://lookaside.fbsbx.com/file/Book%20I_Consolidated%20C…hqsTJHSGnedWZ0OSOpxZmrtUVtu6RtpxVMMpL7Kt2IHuRsCvg 22/10/2018, 9W25 PM


Page 27 of 90
all under his supervision. Patently, there was no further threat to the life and limb of
accused.

The Court agreed with the Sandiganbayan that the petitioner failed to prove the defense
he had raised. The primordial requisite of self-defense is unlawful aggression. And for
unlawful aggression to be present, there must be a real danger to life or personal safety. In
the instant case, there was no imminent and real danger to the life or limb of the
petitioner when he shot the deceased, since the latter had already been disarmed. As
former Chief Justice Aquino states in his book on Criminal Law: “In order to justify self-
defense, it is essential that the attack upon defendant be simultaneous with the killing, or
preceded the latter without an appreciable interval of time. (Ferrer, 1 Phil. 56), x x x The
harm caused by one person to another who offended or caused him injury, sometime after
he suffered such offense or such injury, does not constitute an act of self-defense, but an
act of revenge. Absent the element of unlawful aggression, there is no self-defense
complete (Art. II,par. 1) or incomplete (Art. 13, par. 1, RPC).

#23
SYCIP JR. VS COURT OF APPEALS

​Bouncing checks Law (BP No. 22), an offense is committed


when the following elements are present: (1) The making, drawing
and issuance of any check to apply for account or for value; (2) the
knowledge of the maker, drawer, or issuer that at the time of the issue
he does not have sufficient funds in or credit with the drawee bank for
the payment of such check in full upon in present merit; and (3) the
subsequent dishonour of the check by the drawee bank for
insufficiency of funds or credit of dishonour for the same reason had

https://lookaside.fbsbx.com/file/Book%20I_Consolidated%20C…hqsTJHSGnedWZ0OSOpxZmrtUVtu6RtpxVMMpL7Kt2IHuRsCvg 22/10/2018, 9W25 PM


Page 28 of 90
not the drawer, without any valid cause, ordered the bank to stop
payment. BP 22 creates a presumption juris tantum that the second
element prima facie exists when the first and third elements of the
offense are present but such evidence may be rebutted, if not rebutted
or contradicted. It will suffice to sustain a judgment in favour of the
issue, which it supports.

​Postdating simply means that on the date indicated on in face,


the check would be properly funded not that the checks should be
deemed as issued only then. Closure of petitioners account was made
upon the advice of the drawee bank, to avoid payment of hereby bank
charges each time petitioner issued a “stop payment” order to prevent
encashment of post-dated checks in private respondent’s possession
what the prosecution has established is the closure of petitioners
checking account. But this does not suffice to prove the second
element of the offense under BP bilang 22, which explicitly “evidence
of knowledge or insufficiency of funds” by the accused at the time the
checks are presented for encashment. It must be stressed that every
element of the offense must be proved beyond reasonable doubt,
never presumed.

Moreover,​ protection must also be afforded the interest of


townhouse buyers under P.D no. 957. A statute must be construed in
relation to other laws so as to carry out the legitimate ends and
purposes ends and purpose intended by the legislative. Courts will not
strictly follow the letter of one statute when it leads away from the
true intent of legislative and when its ends an inconsistent with the
general purpose of the act, more so, when it will mean the
contravention of another valid statute. Both laws have to be
reconciled and given due effect.

​Section 23 of PD 957 to suspend payments until such time



as the owner or developer had fulfilled in obligations to the buyer.
This exercise of a statutory right to suspend instalment payments is a
valid defense against the purported violation of BP bilang 22 that
petitioner is charged with to say the least, The third element of
“subsequent dishonour of the check without valid cause” appear not
established by the prosecution, then that presumption is unavailing in
the presences of a valid cause to stop payment, hereby negating the
third element of the crime.

​Offenses punished by a special law, like the bouncing checks


law, and not subject to the revised penal code, but the code is
supplementary to such a law.

Facts:

​On August 24, 1989, Francisco T. Sycip agreed to buy on instalment from Frances Realty
Corporation (FRC), a townhouse unit in the latter’s project at Bacoor, Cavite upon
execution of the contract to sell, Sycip, as required issued to FRC, 48 post-dated checks,
each in the amount of P 9,304.00, covering 48 monthly instalments. After moving in his
unit, Sycip complained to FRC regarding defects in the unit and incomplete reactions of
the townhouse project. FRC ignored the complaint. Sycip sent “Stop payment orders” to

https://lookaside.fbsbx.com/file/Book%20I_Consolidated%20C…hqsTJHSGnedWZ0OSOpxZmrtUVtu6RtpxVMMpL7Kt2IHuRsCvg 22/10/2018, 9W25 PM


Page 29 of 90
the bank. When FRC continued to present other checks (postdated) to the bank as the due
date fell. The bank advised Sycip to close his checking account to avoid paying bank
charges every time he made a “stop payment” order on the forthcoming checks. Due to
the closure of petitioners checking account, the drawer bank dishonor’s six postdated
checks. FRC filed a complaint against petitioner for violations of Bp bilang 22 involving
said dishonoured checks.

#24
PEOPLE V DANSAL
G.R. NO. 105002, July 17, 1997

FACTS:
Dansal, armed with a Garand rifle like his four (4) other companions, fired fatal shots
which caused the death of Alamat. Witnesses, Antalo and Mosa pointed out the appellant
as the perpetrator of the crime. According to them, they saw the victim, conversing with
five (5) persons, one of whom was Dansal. When they were five (5) meters away from
the group, they heard gunshots. Antalo turned his head and noticed that a smoke was
coming out of appellant's rifle and empty shells were falling therefrom. The Garand rifles

https://lookaside.fbsbx.com/file/Book%20I_Consolidated%20C…hqsTJHSGnedWZ0OSOpxZmrtUVtu6RtpxVMMpL7Kt2IHuRsCvg 22/10/2018, 9W25 PM


Page 30 of 90
of Dansal and his companions were pointed at the victim who was lying on the ground.
Dansal claims exemption from criminal liability under Article 12, paragraph 5 of the
Revised Penal Code, because he allegedly acted under the compulsion of an irresistible
force. He allegedly joined the armed Dorados against his will because of fear for his own
safety.
The trial court concluded that the offenders consciously and deliberately adopted the
particular means, method or form of attack employed by them to ensure the
accomplishment of their purpose with impunity. Thus, treachery, abuse of superior
strength and evident premeditation were appreciated in the conviction of the appellant.
ISSUES:
1. Whether or not Dansal must be exempted from criminal liability.

2. Whether or not treachery, abuse of superior strength and evident premeditation


must be appreciated in the conviction of the Dansal.

HELD:
1. Negative. A person who invokes the exempting circumstance of compulsion due to
irresistible force must show that the irresistible force reduced him to a mere
instrument that acted not only without will but also against his will. The
compulsion must be of such character as to leave the accused no opportunity to
defend himself or to escape. The duress, force, fear or intimidation must be
present, imminent and impending; and it must be of such a nature as to induce a
well-grounded apprehension of death or serious bodily harm if the act is not done.
A threat of future injury is not enough.

In this case, appellant failed to show such compulsion. In his testimony, he did not
mention that the Dorados physically or morally threatened to kill or hurt him. He
did not even make any attempt to resist. He simply took for granted that they
would kill or hurt him if he did not allow them. In other words, appellant failed to
prove that the Dorados made a real and imminent threat on his life or climb
sufficient to overcome his free will.

2. Affirmative as to treachery; Negative as to abuse of superior strength and


evident premeditation. The evidence of the prosecution adequately established
only treachery. The victim knew that appellant and his companions were carrying
those rifles but instead, the victim stayed and spoke with them. The victim,
therefore, had no idea that he was going to be shot by appellant who, after all, was
his relative. The victim — being alone and unarmed — could not have defended
himself against all five of them.

Abuse of superior strength, on the other hand, was not established, as there was
no testimony to the effect that appellant and his companions took advantage of
their collective strength in order to kill the victim. Mere superiority in number
after all is not necessarily indicative of this aggravating circumstance.

The prosecution also failed to establish evident premeditation. For this qualifying
circumstance to be appreciated, there must be a lapse of sufficient time to afford full
opportunity for meditation and reflection that would allow the conscience of the actor to
attempt to overcome the resolution of his will. The prosecution was unable to establish
this time element as its evidence dealt merely with the circumstances of the actual
shooting itself.

https://lookaside.fbsbx.com/file/Book%20I_Consolidated%20C…hqsTJHSGnedWZ0OSOpxZmrtUVtu6RtpxVMMpL7Kt2IHuRsCvg 22/10/2018, 9W25 PM


Page 31 of 90
#25
People of the Philippines vs. Sazon
G.R. No. 89684, September 18, 1990.

Topics: Self-defense

Facts:

Wilfredo Longno, the victim, was allegedly followed by Gerardo Sazon (the accused) and
Cornelio Altejos. When Longno reached the bench near the public faucet, the accused
pointed a gun at him and said “What are you going to do?” Longno replied, “brod, just
shoot.” Irked by the response, the accused fired the gun hitting Longno in the left
forearm. The accused and Longno then grappled for the gun. It was while the two were
thus struggling that Altejos stabbed Longno in the chest, after which both the accused and
Altejos ran away. Longno was rushed to the hospital but he did not survive. Sazon was
then charged with murder. In the trial, he claims that he only acted in self-defense. He
claims that the deceased had a revolver tucked in his waist and was about to draw the
same. He therefore parried the gun and it was then he pulled out his gun and shot Longo

https://lookaside.fbsbx.com/file/Book%20I_Consolidated%20C…hqsTJHSGnedWZ0OSOpxZmrtUVtu6RtpxVMMpL7Kt2IHuRsCvg 22/10/2018, 9W25 PM


Page 32 of 90
in the forearm. The trial court convicted the accused with murder as the killing was
attended by evident premeditation.

Issue:

Whether or not there was a valid self-defense?

Held:

No. It is a statutory and doctrinal requirement that for the justifying circumstance of self-
defense, the presence of unlawful aggression is a condition sine qua non. There can be no
self-defense, complete or incomplete, unless the victim has committed an unlawful
aggression against the person defending himself. In this case, the defense failed to
establish the primary element of unlawful aggression on the part of the victim and,
therefore, the plea of self-defense must fail. Aside from the inconsistent testimonies of
the accused, unlawful aggression on the part of the victim is further negated by the
physical evidence in the case. The paraffin test conducted on the cadaver of the deceased
showed that the hands of the deceased were negative for gunpowder residues indicating
that he did not fire a gun during the incident. The other parts of his body like his forearm
and his abdomen bore strong traces of gunpowder residues because of the burst of the
gun of the accused.

#26
People vs Roxas

Facts:
Accused Roxas was charged with homicide for inflicting upon Felicisimo Garcia two
wounds in different parts of the body with an automatic pocket-knife, the aforesaid
Garcia died instantaneously.

The defense contended that the act of Roxas in stabbing the deceased was made in self-
defense, a justifying circumstance as prescribed in Article 11 Section 1 of the Revised
Penal Code. However, Roxas admitted that the deceased was not armed during the
aggression. In addition, defense contended that Roxas was a minor at the time of the
commission of the crime. Thus, warranting application of mitigating circumstance of
minority as provided for in Article 13 Section 2.

The Regional Trial Court convicted Roxas for the crime of homicide and he was
sentenced to twelve years and one day of reclusion temporal and required to indemnify
the heirs of the deceased.

Issue:
Whether Roxas’ justifying circumstance of self defense shall be appreciated in his favor
despite an unarmed aggressor and whether mitigating circumstance of minority shall
apply in the instant case.

https://lookaside.fbsbx.com/file/Book%20I_Consolidated%20C…hqsTJHSGnedWZ0OSOpxZmrtUVtu6RtpxVMMpL7Kt2IHuRsCvg 22/10/2018, 9W25 PM


Page 33 of 90
Held:
No self-defense under Article 11 Section 1 was made. Even though the deceased was the
aggressor, the defendant himself admits that the aggressor was not armed. There being no
rational necessity shown for the means which the accused used to repel the attack of his
aggressor, article 11 of the Revised Penal Code cannot be invoked here to exempt the
defendant from the responsibility.

Prosecution states that the age of the accused at the time of the commission of the offense
was seventeen years and eight months. The Court accepts this as the best evidence in the
record as to the true age of the accused. It is necessary, therefore, to reform the judgment
of the court below and to enter one in conformity with article 80 of the Revised Penal
Code, as what Article 13 Section 2 provides.

Moreover, the Court found the following extenuating circumstances and direct that they
be applied when final sentenced may be pronounced, namely: First, the age of the
accused being under eighteen (article 13, paragraph 2); second, provocation on the part of
the deceased (article 13, paragraph 4); third, obfuscation (article 13, paragraph 6); fourth,
voluntary surrender (article 13, paragraph 7).

Hence, penalty imposed by the trial court was revoked and Roxas was placed in the
Philippine Training School for Boys at Welfareville in the custody of the care of the
Commissioner of Public Welfare until the said defendant shall have attained his majority.

#28
THE PEOPLE OF THE PHILIPPINES vs. BENJAMIN GONDAYAO, alias
"BEN", ET AL.
G.R. No. L-26240 | October 31, 1969
CONCEPCION, C.J.:

DOCTRINE / RULING:
It is clear that Benjamin stabbed Piol twice from behind, after disarming him.
Considering moreover that Benjamin has provoked the incident, by hurling
uncomplimentary remarks at his political opponents, one of whom was Piol; that such
remarks led to an altercation with Piol, in consequence of which, stones were thrown at
him, hitting him on the head; that when, owing to the impact of said stone, which could
have rendered him groggy, and the lacerated injuries thus sustained by him, Piol prepared
himself to fight by drawing out his dagger, Benjamin accepted the challenge resulting
from this act, by "rushing" to his encounter and grappling with him; and that, accordingly,
Benjamin cannot be given the benefit of either complete or incomplete self-defense.
Although Piol was stabbed from behind, Benjamin did not act with treachery, for this was
merely an incident of their struggle, which had begun with both contenders facing each
other, each prepared for the fight that ensued.

FACTS:
After grappling with Benjamin Gondayao, near the market place, in the Barrio of Paitan,
Municipality of Sual, Province of Pangasinan, on November 14, 1965, Orlando Piol
appeared to have, in addition to several lacerations on the head, two stab wound son the
back in consequence of which he died that same afternoon, due to massive internal
hemmorrhage. Immediately after the occurrence, Benjamin assumed full responsibility
for said injuries, which he claimed to have inflicted in self-defense. On November 17,
1965, Macario Aquino, Chief of Police of Sual, filed, with the Municipal Court thereof, a
complaint charging Benjamin Gondayao with homicide. Twelve days later, Aquino filed
an amended complaint charging murder, allegedly committed, not only by Benjamin

https://lookaside.fbsbx.com/file/Book%20I_Consolidated%20C…hqsTJHSGnedWZ0OSOpxZmrtUVtu6RtpxVMMpL7Kt2IHuRsCvg 22/10/2018, 9W25 PM


Page 34 of 90
Gondayao, but also by his brother Anoy Gondayao, and Eduardo Bersamina, Eto
Agbayani and Jovencio Yanday. Soon thereafter, the correspondinginformation for
murder was filed, with the Court of First Instance of Pangasinan, against said five
defendants.

ISSUE:
Whether or not the theory of self-defense can be appreciated in favor of Benjamin
Gondayao.

#29
ELIAS VALCORZA, petitioner, vs. PEOPLE OF THE PHILIPPINES, respondent.
G.R. No. L-28129 ​October 31, 1969

TOPIC: Performance of official duty as a justifying circumstance

FACTS:
A detention prisoner who was charged with a relatively minor offense, namely, stealing a
chicken, escaped from his detention cell. When ordered to stop by the police, he ran away
and then threw himself into a creek to elude his pursuer. He suddenly emerged from
bushes near the accused and a fellow policeman and assaulted the accused policeman
twice with a stone and then ran away again. Believing that the escaping detainee would
be able to elude him and his companions, the accused after a chase, decided to fire five
cautionary shots into the air and then aimed directly at the Escaping detainee. Accused
was charged with homicide.

ISSUE:
Whether accused should be acquitted of the charge of homicide for the reason that the act
was done in the performance of his duty

RULING:
Yes, accused policeman is must be acquitted from the charge of homicide. The act thus
performed by accused was committed in the performance of his official duty and was
more or less necessary to prevent the escaping prisoner from successfully eluding the
officers of the law. To hold him guilty of homicide may have the effect of demoralizing
police officers discharging official functions identical or similar to those in the
performance of which accused was engaged when he fired at the escaping detainee.

https://lookaside.fbsbx.com/file/Book%20I_Consolidated%20C…hqsTJHSGnedWZ0OSOpxZmrtUVtu6RtpxVMMpL7Kt2IHuRsCvg 22/10/2018, 9W25 PM


Page 35 of 90
#30
PEOPLE vs. AVELINO MANANSALA, JR., ET AL.
G.R. No. L-23514
February 17, 1970
This is an appeal from the decision of the Court of First Instance of Albay in its Criminal
Case No. 3285 convicting Avelino Manansala, Jr. and Jose Manansala of murder and
sentencing the first to an indeterminate penalty of from 10 years and 1 day of prision
mayor to 17 years 4 months and 1 day of reclusion temporal, and the second to reclusion
perpetua.
FACTS:
In the afternoon of March 27, 1962, some persons had a quarrel inside a Carinderia. Jose
Manansala had Rodrigo in a tight embrace from behind and Avelino stabbed the latter
with a balisong. When the policemen arrived they found Avelino still holding the fatal
weapon, and Rodrigo's limp body, bathed in his own blood, stretched on the papag.
While being taken to a hospital, Rodrigo died. The autopsy revealed that the victim died
from massive hemorrhage and shock as a result of the multiple wounds (thirteen in all)
sustained by him in the chest, abdomen, back and the extremities.
The defense witnesses testified that it was Rodrigo who first attacked Avelino in the face
due to argument on the payment for his services in carrying the latter’s baggage. After the
third blow, Avelino fell down and a batangas knife he was carrying fell from his pocket.
Rodrigo, seeing that Avelino was armed rushed to the carinderia and returned with a 10-
inch knife which he swung at Avelino but the latter evaded the blow.
Meanwhile,Jose Manansala, an uncle of Avelino, noticed the commotion from outside
the carinderia and shouted at Rodrigo to stop. Rodrigo instead delivered another thrust at
Avelino, who again evaded it. At the third attempt, Jose embraced Rodrigo from behind,
and it was at that moment that Avelino stabbed Rodrigo and inflicted the numerous
wounds which proved fatal
The trial court found both accused guilty of murder with the mitigating circumstance of
sufficient provocation or threat on the part of the victim, and imposed upon him a lesser
penalty than that imposed on his uncle.
ISSUE:
Whether or not the penalty imposed by the trial court is proper.
HELD:

https://lookaside.fbsbx.com/file/Book%20I_Consolidated%20C…hqsTJHSGnedWZ0OSOpxZmrtUVtu6RtpxVMMpL7Kt2IHuRsCvg 22/10/2018, 9W25 PM


Page 36 of 90
Yes. The slaying of the deceased having been qualified by treachery, Avelino Manansala
is liable for murder, the penalty of which is reclusion temporal maximum to death (Art.
248, Revised Penal Code). Appreciating in favor of Avelino the mitigating circumstance
of sufficient provocation by the deceased without any generic aggravating circumstance
to offset the same, the penalty imposable upon him is the minimum period of the penalty
for murder (see par. 3, Art. 63, Revised Penal Code), which is reclusion
temporal maximum (17 years, 4 months and 1 day to 20 years). Since the resulting
penalty is neither death nor life imprisonment, the Indeterminate Sentence Law applies
(Sec. 2, Act No. 4103 as amended).
Avelino Manansala is therefore entitled to an indeterminate sentence, the upper range of
which is reclusion temporal maximum and the lower range — which is one degree lower
than the penalty prescribed by the Revised Penal Code for murder — is anywhere
within prision mayor maximum (10 years and 1 day) to reclusion temporal medium (17
years and 4 months).
The penalty meted out by the trial court on Avelino Manansala, Jr. — "from 10 years and
1 day of prision mayor to 17 years 4 mouths and 1 day of reclusion temporal" — is
within the range allowed by law and is therefore correctly imposed.
As regards appellant Jose Manansala, the penalty prescribed by law, he being an
accomplice, is one degree lower than that prescribed for the principal, or prision
mayor maximum to reclusion temporal medium (10) years and 1 day to 17 years and 4
months). Applying the Indeterminate Sentence Law, and considering that under
paragraph 1, Article 64 of the Revised Penal Code, "(W)hen there are neither aggravating
nor mitigating circumstances, (the court) shall impose the penalty prescribed by law in its
medium period," the decision appealed from should be, as it is hereby, modified as
follows:
Appellant Jose Manansala, as accomplice to the offense of murder, is sentenced to an
indeterminate penalty of from 4 years, 2 months and 1 day of prision correccional to 12
years and 1 day of reclusion temporal; the amount of the indemnity is raised from
P6,000.00 to P12,000.00 to be paid the heirs of the deceased by Avelino Manansala, Jr. as
principal; in case of the insolvency of the principal, Jose Manansala, as accomplice, is
subsidiarily liable for the indemnity due from said principal; and in all other respects the
judgment appealed from is affirmed. No costs in this instance.

#31
G.R. No. L-32390. December 28, 1973
THE PEOPLE OF THE PHILIPPINES THE PEOPLE OF THE PHILIPPINES, p
lain tif f- a p p elle e , v s . v s . CLEMENTE CLEMENTE AQUINO AQUINO, d e f
e n d a n t- a p p ella n t .

https://lookaside.fbsbx.com/file/Book%20I_Consolidated%20C…hqsTJHSGnedWZ0OSOpxZmrtUVtu6RtpxVMMpL7Kt2IHuRsCvg 22/10/2018, 9W25 PM


Page 37 of 90
Facts:
Clemente Aquino was charged with and found guilty of murder for the death of Pedro
Cruz. Cruz was an employee of the Farmont Mines and Aquino was working as truck
driver for a certain Leoncio de Guzman. Due to rivalry between the two mining
establishments, which somehow involved their respective personnel, Aquino shot Cruz.
Aquino interposed self defense. He did not drive thru or ram the fence but he accidentally
rammed three or four bamboo posts which were stuck into the ground. He proceeded on
the dry river bed, stopped to unload the soldier and his family near their hut, and then
continued on his way. He had covered a short distance when he saw a jeep speeding in his
direction, with Pedro Cruz at the wheel and a companion, called "Moro," beside him.
Cruz blocked his way with the jeep, got off and walked towards the left side of the truck.
About two meters away Cruz stopped and asked him, "Why do you insist passing here
when I am stopping you?" Aquino answered, "Why should you stop me here when I am
already on the river?" Cruz did not answer, but gave a meaningful dance at "Moro," who
was still inside the jeep, and touched something bulging at his waist. When he pulled the
thing out, Aquino saw it was a "balisong" knife. Cruz thrust forward with his knife at
him. Aquino got his gun from the tool box and fired at his antagonist. The first shot hit
Cruz on the right side of the throat, the second on the right arm. Aquino fired four times
in all.

Issue: W/N self defense is meritorious.

Ruling:

Yes.

The physical, objective facts enumerated above are not only consistent with but indeed
confirm strongly the plea of self-defense raised by the appellant. The direction of three of
the four bullets which hit the deceased shows that he must have been in a forward
stooping position at the time, with his left forearm raised somewhat in front of him, as
would be the case if he was holding the windshield frame with his left hand. Thus the
slug which entered the base of the neck, in front and to the right, plowed downward
through the upper lung and the muscle of the lumbar region, where it was recovered just
beneath the skin. The slug which entered the body at the left upper chest also followed a
downward direction and was recovered beneath the skin at the back. And the slug which
hit the left forearm near the palm of the hand took both an upward and posteriorly
direction and exited "at the proximal 3rd of the forearm medially." The bullets could not
have had these trajectories if the deceased had been standing upright two or three meters
to the left of the truck, as the witnesses for the prosecution testified. The evidence of the
dagger or knife, which was retrieved by patrolman Salazar from the floor of the truck
below the driver's seat, and the evidence of the blood stains on the seat itself, not only
find no explanation in but directly contradict the version of the prosecution. The very
position of the body as testified to by the same patrolman and confirmed by the
photographs shows that after being shot Cruz must have slid down the running board of
the truck where he was said to be standing, thus accounting for the fact that he was found
lying on his back with his two legs under the truck. Such a position could hardly have
been possible if he had been shot while standing two or three meters away from the
vehicle.

The three elements of self-defense are here present. There was unlawful aggression on
the part of the deceased when he attacked the appellant with a deadly weapon. The means
employed to repel that aggression was reasonable. As repeatedly held by this Court, "(I)n
emergencies of this kind, human nature does not act upon process of formal reason but in
obedience to the instinct of selfpreservation; and when it is apparent that a person has

https://lookaside.fbsbx.com/file/Book%20I_Consolidated%20C…hqsTJHSGnedWZ0OSOpxZmrtUVtu6RtpxVMMpL7Kt2IHuRsCvg 22/10/2018, 9W25 PM


Page 38 of 90
reasonably acted upon his instinct, it is the duty of the courts to sanction the act and hold
the act irresponsible in law for the consequences." 1 1 And upon a review of the evidence
we find that the appellant did not give any sufficient provocation for the attack against
him.

#32
People v Encomienda
GR no L-26750 Aug 18 1972
Topic: Self-defense

Facts:
Cabaral, the hacienda supervisor, tried to evict encomienda, a tenant. When Encomienda
resisted by pleading with Cabaral and telling the latter that he had no right, Cabaral drew
his gun. Encomienda was able to hold the hand that drew the gun and pin Cabaral to the
ground. During such, Cabaral let go off the gun. Thereafter, while Cabaral was trying to
reach the gun, Encomienda hacked Cabaral’s both hands. When Cabaral persisted,
Encomienda hacked Cabaral’s head killing him.

https://lookaside.fbsbx.com/file/Book%20I_Consolidated%20C…hqsTJHSGnedWZ0OSOpxZmrtUVtu6RtpxVMMpL7Kt2IHuRsCvg 22/10/2018, 9W25 PM


Page 39 of 90
Issue:
Whether or not Encomienda is entitled to the Justifying circumtances of self-defense.

Held:
Encomienda is entitled to the Justifying circumtances of self-defense. All the requisites
are present. There was unlawful aggression when Cabaral drew his gun. The hacking by
the bolo was reasonably necessary as Cabaral kept reaching for the gun. And there was
no finding that Encomienda provoked Cabaral.

#33
C.A. No. 384 February 21, 1946
THE PEOPLE OF THE PHILIPPINES, plaintiff-appellee, vs. NICOLAS
JAURIGUE and AVELINA JAURIGUE, defendants. AVELINA JAURIGUE,
appellant.

Facts:

https://lookaside.fbsbx.com/file/Book%20I_Consolidated%20C…hqsTJHSGnedWZ0OSOpxZmrtUVtu6RtpxVMMpL7Kt2IHuRsCvg 22/10/2018, 9W25 PM


Page 40 of 90
Avelina Jaurigue and Amado Capiña lived in the barrio of Sta. Isabel, City of San Pablo,
Province of Laguna. Amado had been courting Avelina. On one occasion, Amado
approached her and confessed his love. He also suddenly embraced, kissed, and touched
her breasts. She slapped him, gave him a fist blows and kicked him. Since then, she
armed herself with a long fan knife. Thereafter, Amado also climbed up the house of the
defendant and entered the room while Avelina was sleeping. He felt her forehead,
evidently with the intention of abusing her. She screamed for help, which awakened her
parents. Amado came out of the place where he was hiding, under the bed and kissed the
hand of Nicolas Jaurigue, asking for forgiveness. Nicolas sent for the barrio lieutenant
and Amado’s parents. Amado’s parents apologized for their son’s misconduct. After
which, Avelina received information that Amado had been falsely boasting in the
neighborhood of having taken liberties with her person and asked Amado to elope with
her and if he wouldn’t marry her, she’ll poison herself.

At the end of the same day, defendant and appellant Avelina Jaurigue entered the chapel
for the purpose of attending religious services, and sat on the bench next to the last one
nearest the door. Amado Capina, deceased, was seated on the other side of the chapel.
Upon observing the presence of Avelina Jaurigue, Amado Capina went to the bench on
which Avelina was sitting and sat by her right side, and, without saying a word, Amado,
with the greatest of impudence, placed his hand on the upper part of her right thigh. On
observing this highly improper and offensive conduct of Amado Capina, Avelina
Jaurigue, conscious of her personal dignity and honor, pulled out with her right hand the
fan knife, which she had in a pocket of her dress, with the intention of punishing Amado's
offending hand. Amado seized Avelina's right hand, but she quickly grabbed the knife
with her left hand and stabbed Amado once at the base of the left side of the neck,
inflicting upon him a wound, which was necessarily mortal. Barrio lieutenant Casimiro
Lozada, who was also in the same chapel, approached Avelina and asked her why she did
that, and Avelina surrendered herself, saying: "Kayo na po ang bahala sa aquin,"
meaning: "I hope you will take care of me," or more correctly, "I place myself at your
disposal." Fearing that Amado's relatives might retaliate, barrio lieutenant Lozada
advised Nicolas Jaurigue and herein defendant and appellant to go home immediately, to
close their doors and windows and not to admit anybody into the house, unless
accompanied by him. Avelina Jaurigue was found guilty of homicide for killing Amado
Capiña by Court of First Instance of Tayabas.

ISSUE: WON respondent should be completely absolved of criminal liability


considering her act done in defense of her honor.

Held: No. Avelina Jaurigue is not exempted from criminal liability. The means employed
by her in the defense of her honor was evidently excessive and she cannot be legally
declared completely exempt from criminal liability. With the modification of the
judgment appealed from, Avelina is sentenced to indeterminate penalty ranging from 2
months and 1 day arresto mayor minimum to 2 years 4 months and 1 day maximum with
necessary penalties of P2,000.
According to the facts established by the evidence and found by the trial court, when
Amado Capiña (deceased) sat beside the defendant near the chapel door placing his hand
on the upper portion of her right thigh, without her consent, the said chapel was lighted
and there were already several people. Under these circumstances, there was and there
could be no possibility for her to be raped. And so when she struck Capina with a knife
on his neck resulting death, the means she employed to defend her honor was excessive.

https://lookaside.fbsbx.com/file/Book%20I_Consolidated%20C…hqsTJHSGnedWZ0OSOpxZmrtUVtu6RtpxVMMpL7Kt2IHuRsCvg 22/10/2018, 9W25 PM


Page 41 of 90
#34
People of the Philippines vs. Anastacio Apolinar
38 O.G. 28280
July `19, 1938

FACTS:

Midnight of December, Apolinar, armed with a shotgun, was looking over a parcel of
land which he occupies. He suddenly saw a man carrying a bundle of palay on his
shoulder. Apolinar thought it was a thief and tried getting the man’s attention by asking
who the man was. The man ignored him. Apolinar fired in the air and asked again, but to
no avail. The man fleed, which prompted Apolinar to shoot him.

https://lookaside.fbsbx.com/file/Book%20I_Consolidated%20C…hqsTJHSGnedWZ0OSOpxZmrtUVtu6RtpxVMMpL7Kt2IHuRsCvg 22/10/2018, 9W25 PM


Page 42 of 90
The man was later identified as Domingo Petras who was able to return home and
narrate the story to Angel Natividad, the village chief. Petras died because of the wounds
he obtained from the shooting.

Apolinar surrendered to the Justice of Peace, stating that his act was an act of defense
since Petras was going to attack Bonifacio Menndones with a bolo.

ISSUE:

Whether or not Apolinar’s act of shooting Petras was justified by Defense of Property.

HELD:

No. Defense of property is not of such importance as the right to life and it can be
invoked only as a justifying circumstance when it is coupled with an attack on the person
of the one entrusted with the said property.

#35
G.R. No. L-33304 December 13, 1930
THE PEOPLE OF THE PHILIPPINE ISLANDS, plaintiff-appellee, vs.
CONSTANTE SOTELO, ET AL., defendants.
CONSTANTE SOTELO, appellant.
DOCTRINE: Incomplete self-defense with the penalty next below, that is, prision mayor
in its minimum degree, or six years and one day, with the accessories of law.
FACTS:
The Sotelo brothers, namely, Constante, Dominador, and Vicente, were prosecuted in the
CFI of Ilocos Sur for the crime of homicide under the following information: That on or
about the night of December 24, 1929, in the municipality of Narvacan, Ilocos Sur, the

https://lookaside.fbsbx.com/file/Book%20I_Consolidated%20C…hqsTJHSGnedWZ0OSOpxZmrtUVtu6RtpxVMMpL7Kt2IHuRsCvg 22/10/2018, 9W25 PM


Page 43 of 90
said accused Constante, Dominador, and Vicente Sotelo, armed with a penknife, a stick,
and an iron bar, respectively, acting together and helping one another, did willfully,
maliciously, unlawfully, and feloniously with treachery and evident premeditation attack,
beat up, and commit assault upon the person of Ignacio Cambaliza, inflicting a mortal
wound upon him on the level of the left nipple, which penetrated the left lung and the left
ventricle of the heart, another on the outward surface of the right arm, a bruise on the
nose and another on the upper lip: as a result of which said Ignacio Cambaliza died after
a few minutes.
The record shows that at about 8 o'clock in the evening of the 24th of December, 1929,
Ignacio Cambaliza started for the barrio of Ravadabia, in the municipality of Narvacan,
Ilocos Sur, accompanied by Baltazar Capistrano. They took the provincial road leading to
said barrio and as they approached the Sotelo house, they bid the time of day, asking
whether they might pass by, according to the custom of the place. They were barely 20
meters away from the house, when the defendant Constante Sotelo, who had just finished
his supper, descended from the house towards the road, and, from the entrance of his
yard, turned his flashlight on the passers-by to see who they were. When Ignacio
Cambaliza saw this, he walked back to where Constante Sotelo stood and inquired why
he turned his flashlight on them, and what it was he wanted, winding up with a vulgar
remark. When Constante's brothers, who were then in the yard on the side of the road,
saw Cambaliza's attitude, they approached their brother to separate or defend him,
whereupon Cambaliza's commenced beating them with his iron crop, once striking
Constante's arm. The brothers, in turn, fell upon Cambaliza, Dominador striking him
across the face with the stick he carried, and Vicente wounding him in the right shoulder
with a penknife. At this juncture, Capistrano attempted to intervene, but he was warned
by Vicente and probably by Dominador also, for which reason he withdrew from the
scene, and the fight then continued between Cambaliza and Constante. In the course of
this fight, Constante thrust a penknife into Cambaliza at about the level of the left nipple,
producing a wound which penetrated the left lung into the left ventricle of the heart,
resulting in his death a few minute later.
After the trial, the court below found the defendant Constante Sotelo guilty of the crime
of homicide, and the defendants Vicente and Dominador Sotelo of slight physical
injuries, sentencing the former to suffer twelve years and one day of reclusion temporal,
to indemnify the heirs of the deceased in the sum P1,000, with the accessories of law, and
to pay one-third of the costs; and ordering the release of Vicente and Dominador Sotelo
in view of the fact that they had already been imprisoned since December 24, 1929, with
two-thirds of the costs de oficio.
ISSUE:
Whether or not the trial court erred in refusing to consider the plea of self-defense.
HELD:
As to who started the aggression, there is an obvious contradiction between the testimony
of Capistrano and that of the witnesses for the defense. But in the light of sound
judgment, we are inclined to believe that the deceased started the aggression, provoked
by the offensive language used by Constante and his brothers, imputing to him the
utterance of vulgar language against them. In such a situation the deceased naturally used
his whip against those who were in front of him, striking Constante's arm. We therefore
believe this is a case of incomplete self-defense, wherein the appellant was unlawfully
attacked by the deceased and compelled to employ reasonable means to defend himself,
but he is responsible for provoking the attack. (U. S. vs. Ancheta, 1 Phil., 30; U.
S. vs.McCray, 2 Phil., 545.) According to article 86 of the Penal Code the penalty next
below that provided in article 404 of said Code must be imposed upon the appellant, that
is, prision mayor in its minimum degree, or six years and one day, with the accessories of
law, and to indemnify the family of the deceased in the amount of P500. And with this
modification the judgment appealed from is affirmed in all other respects, with costs
against the appellant.

https://lookaside.fbsbx.com/file/Book%20I_Consolidated%20C…hqsTJHSGnedWZ0OSOpxZmrtUVtu6RtpxVMMpL7Kt2IHuRsCvg 22/10/2018, 9W25 PM


Page 44 of 90
#39
People vs. Ong
G.R. No. L-37908 October 23, 1981

FACTS:

Baldomero Ambrosio, the accused-appellant, is one of the four (4) perpetrators in the
information filed for the murder of Henry Chua. The two accused, Benjamin Ong and
Bienvenido Quintos, were already tried and convicted by the trial court at the time of the
arrest of Ambrosio, while the other accused, Fernando Tan, was still at large.

Sometime in April 1971, the four accused took the victim from his Mustang carried him
to the vehicle used by the perpetrators. They all went to Novaliches road, whereby at a
narrow street along the way, they stopped and killed the victim, stabbed twice by Tan
using an ice-pick.

Thereafter, they carried the body and buried it in the hole,which Ambrosio covered.
Ambrosio, claimed that he did not participate voluntarily in the crime, but rather he was
ordered by Tan as to all the acts he did during the execution of the offense.

ISSUE:
1. Whether or not the accused Ambrosio voluntarily participated in the commission of
the crime.
2. Whether or not aggravating and mitigating circumstances are present.
3. Whether Ambrosio should be convicted of murder

RULING:

1. Yes. Quintos, as shown in his testimony, (Exh. "1") clearly narrated how Ambrosio
participated in the crime, manifesting voluntariness in his acts throughout the
execution of the same... He never revealed to the authorities the crime that he
alleged to be an unwilling participant of in that long span of time.

https://lookaside.fbsbx.com/file/Book%20I_Consolidated%20C…hqsTJHSGnedWZ0OSOpxZmrtUVtu6RtpxVMMpL7Kt2IHuRsCvg 22/10/2018, 9W25 PM


Page 45 of 90
During the execution of the felony, Ambrosio never by act or deed protested to
the group regarding its cruel commission. We also cannot fully understand why
Ambrosio had to join the group of Tan, if according to Ambrosio, himself, Tan
was a man of bad reputation. Tan has never been apprehended. The defense of
Ambrosio that he was threatened by Tan to participate in the crime stands
uncorroborated, as contradicted by the testimony of Quintos. The inevitable
conclusion is that Ambrosio voluntarily participated in the commission of the
crime.

2. Yes. As to the aggravating and mitigating circumstances present in the commission


of the crime, this Court already ruled in G.R. No. L-34497, that:
a. Treachery (alevosia) qualified the killing of Chua to murder. Chua's hands
were tied and his mouth was gagged when he was stabbed twice with an ice-
pick. Chua was defenseless and helpless enabling the accused to commit the
crime without risk to them.
b. The aggravating circumstance of abuse of superior strength is absorbed in
treachery.
c. The aggravating circumstance of nighttime (nocturnidad) cannot be absorbed in
treachery because in this crime treachery arose from the defenseless position
of Chua when he was killed, while nighttime was purposely sought by the
accused to facilitate immunity in the commission of the crime.
d. The aggravating circumstance of uninhabited place (despoblado) is also
present, due to the deliberate selection of an isolated place (Barrio Makatipo,
Novaliches, Caloocan City) for killing and burying the victim.
e. Abuse of confidence cannot be considered as an aggravating circumstance
present in the crime, because it does not appear that the victim Chua ever
reposed confidence on Ong. Chua knew that he was far stronger in money
and influence than Ong. The fact that Henry Chua invited Ong night clubbing
on that fatal evening and accommodated the latter in his car did not show that
Chua had confidence in Ong.
f. The aggravating circumstance of use of motor vehicle in the commission of the
crime can be considered present because the Biscayne car of Ong was used to
trail the victim's car and to facilitate the commission of the crime and the
escape of the accused.
g. Cruelty (ensañamiento) cannot be considered because there is no evidence that
the victim Chua was buried while still alive to make him suffer.
h. Evident premeditation attended the commission of the crime because the
accused meditated, planned, and tenaciously persisted in the accomplishment
of the crime.

Accused Ong was given the mitigating circumstances of plea of guilty and one
analogous to passion and obfuscation because Chua previously threatened Ong for
non-payment of debt arising from gambling, causing Ong humiliation and shame.

Yes. That the crime of murder was committed has already been established by the
prosecution beyond reasonable doubt when this Court convicted Ong and Quintos in G.R.
No. L-34497. This Court already ruled that "Conspiracy, connivance and unity of
purpose and intention among the accused were present throughout in the execution of this
crime. The four participated in the planning and execution of the crime and were at the
scene in all its stages. They cannot escape the consequences of any of their acts even if
they deviated in some detail from what they originally thought of. Conspiracy implies
concert of design and not participation in every detail of the execution. Thus, treachery
should be considered against all persons participating or cooperating in the perpetration
of the crime."

https://lookaside.fbsbx.com/file/Book%20I_Consolidated%20C…hqsTJHSGnedWZ0OSOpxZmrtUVtu6RtpxVMMpL7Kt2IHuRsCvg 22/10/2018, 9W25 PM


Page 46 of 90
#40
PEOPLE OF THE PHILIPPINES, Plaintiff-Appellee,
vs.RICKY ARGUTA alias "JOEL" and WILSON CAHIPE alias "SIWIT,"
Accused-Appellants.
G.R. No. 213216 ​April 20, 2015

TOPIC: QUALIFYING CIRCUMSTANCES


The presence of 2 qualifying circumstances - "use of a deadly weapon" or "by two or
more persons" - qualifies the crime. If one is present, the remaining circumstance, if also
attendant, is not a generic aggravating circumstance.

FACT: On December 1996, the victim AAA was asked by her father to fetch her sister in
school. Not finding her sister, she decided to go back home instead. There she met the
two defendants Arguta and Cahipe. She was threatened by the two with bladed knife and
forced her to a cottage were she have carnal knowledge that is against her will. In their
defense, both denied said claim of rape and provided their separate alibis. RTC convicted
the two with Simple Rape despite the fact that two qualifying circumstances are present:
(1) use of a deadly weapon and (2) that two persons committed the rapes. . CA affirmed
conviction, thus, the present appeal.

ISSUE: WON the presence of two qualifying circumstances converts one into an
aggravating circumstance

RULING:
No. The presence of either circumstance - "use of a deadly weapon" or "by two or more
persons" - qualifies the crime. If one is present, the remaining circumstance, if also
attendant, is not a generic aggravating circumstance. When the two circumstances are
present, there is no legal basis to consider the remaining circumstance as a generic
aggravating circumstance for either is not considered as such under Article 14 of the
Revised Penal Code enumerating what are aggravating circumstances. Pursuant thereto,
accused-appellants should be sentenced with the penalty of reclusion perpetua, without
eligibility for parole.

https://lookaside.fbsbx.com/file/Book%20I_Consolidated%20C…hqsTJHSGnedWZ0OSOpxZmrtUVtu6RtpxVMMpL7Kt2IHuRsCvg 22/10/2018, 9W25 PM


Page 47 of 90
#42
People of the Philippines vs. Armando Regala
G.R. No. 130508
April 5, 2000

FACTS:

Nerissa Tagala, 16, and her grandmother, Consuelo Arevalo were asleep in their house
one night in September. Unknown to the two, Armando Regalo along with two other
companions got inside the house through the kitchen. The suspects went to where the two
victims were sleeping, poked a gun on them, and tied them. Besides taking 3,000 pesos
and 2 rings worth 6,000 from Consuelo’s closet, Armando also raped Nerissa while the
latter was tied twice.

Armando was later arrested and interposed an alibi. He claimed that he was staying 5
kilometers from the victim’s place. He claims that he was staying at the house of the
manager where he was employed. Armando’s alibi, however, was too weak and was
convicted at the trial court. Armando appealed to the Supreme Court, contending that the
trial court erred in finding him guilty beyond reasonable doubt of robbery with rape.

ISSUE:

Whether or not additional rape can be considered as an aggravating circumstance when


committed in a crime of robbery.

HELD:

No. It should be noted that there is no law providing that the additional rape/s or
homicide/s should be considered as aggravating circumstance. The enumeration of
aggravating circumstances under Article 14 of the Revised Penal Code is exclusive as
opposed to the enumeration in Article 13 of the same code regarding mitigating
circumstances where there is a specific paragraph (paragraph 10) providing for analogous
circumstances.
It is true that the additional rapes (or killings in the case of multiple homicide on the
occasion of the robbery) would result in an “anomalous situation” where from the
standpoint of the gravity of the offense, robbery with one rape would be on the same level
as robbery with multiple rapes. However, the remedy lies with the legislature. A penal
law is liberally construed in favor of the offender and no person should be brought within
its terms if he is not clearly made so by the statute.
In view of the foregoing, the additional rape committed by herein accused-appellant
should not be considered as aggravating. The penalty of reclusion perpetua imposed by
the trial court is proper.

#44
People vs Berdida GR L – 20183
June 30, 1966
Facts
At about 10 o'clock in the evening of 7 May 1960, Antonio Maravilla, Federico Cañalete,
Virgilio Haban and Pedrito Rapadas left the store of one Mang Terio at Mabuhay Street,

https://lookaside.fbsbx.com/file/Book%20I_Consolidated%20C…hqsTJHSGnedWZ0OSOpxZmrtUVtu6RtpxVMMpL7Kt2IHuRsCvg 22/10/2018, 9W25 PM


Page 48 of 90
North Harbor, Tondo, Manila, and proceeded walking towards their homes. They were
met on their way by Eduardo Berdida, Antonio Louie, one Tiquio and one alias Ifugao,
who identified themselves as detectives, told them not to move, and pointed sharp and
long bolos to them. Antonio Louie then dealt a fist blow on Antonio Maravilla. After that,
the group took Antonio Maravilla and Federico Cañalete along the rail tracks, telling
them that they had done something wrong.

At the end of the rail tracks, said group tied the hands of Antonio Maravilla and Federico
Cañalete. After doing this, they dragged the two and took them to a place in Pier 8 at the
North Harbor.

Antonio Maravilla's sister, Elizabeth the help of Patrolman Carlos Pili, who was then at
the corner of Kaguitingan and Lakandula Streets in front of Pier 6 So they proceeded
further, entering a small alley. As they went on, Elizabeth found the shoes of her brother.
Elizabeth, together with Patrolman Pili, returned and went further to the interior of
Mabuhay Street. Finally, they came upon Federico Cañalete and Antonio Maravilla,
sprawled on the ground, the former face down, the latter flat on his back. Federico
Cañalete was found dead. Antonio Maravilla was alive, though his face was swollen,
rendering him barely recognizable. Antonio Maravilla was taken to the North General
Hospital. Patrolman Pili, meanwhile, went still further to the interior and saw, about 12
meters away from where they found the victims, a group drinking liquor. At the approach
of Patrolman Pili, about four men ran away, leaving behind four men, namely, Loreto
Saberon, Mario Mustrado, Cristoto Mitilla and Protacio Libres, the last mentioned being
then drunk and asleep on a bamboo bed. A Mobile Patrol car thereafter arrived and
apprehended them, except Libres. Patrolman Pili next went towards a house near
Tagumpay Street in which direction the others had fled. In said house, which was that of
Crisanta Melgar, the patrolman found some persons who pretended to be sleeping,
namely, Demetrio Garin, Jesus Felicia and Eduardo Berdida. Patrolman Pili brought them
outside and they were taken by the Mobile Patrol to the Detective Bureau

Appellants would, first of all, assail Antonio Maravilla's testimony identifying them as
the assailants, for the reason that he lost consciousness, and, therefore, could not be relied
upon to make said identification and insist on the defense of alibi. that before Maravilla’s
sense faded out he saw herein appellants perform their atrocities on himself as well as on
Federico Cañalete

Issue:

Whether or not the aggravating circumstances of nighttime, abuse of superior strength,


and evident premeditation should be fixing the penalty.

Decision of the court:

The presence of one generic aggravating circumstance, apart from the qualifying
circumstance of treachery, suffices to fix the penalty for murder at the extreme
punishment of death. For there is no mitigating circumstance in the present case. From
the facts and evidence of record in this case, it is clear that appellants took advantage of
nighttime in committing the felonies charged. For it appears that to carry out a sentence
they had pronounced upon Antonio Maravilla and Federico Cañalete for the death of one
Pabling, they had evidently chosen to execute their victims under cover of darkness, at
the dead of night, when the neighborhood was asleep. Inasmuch as the treachery
consisted in the fact that the victims' hands were tied at the time they were beaten, the
circumstance of nighttime is not absorbed in treachery, but can be perceived distinctly

https://lookaside.fbsbx.com/file/Book%20I_Consolidated%20C…hqsTJHSGnedWZ0OSOpxZmrtUVtu6RtpxVMMpL7Kt2IHuRsCvg 22/10/2018, 9W25 PM


Page 49 of 90
therefrom, since the treachery rests upon an independent factual basis. A special case
therefore is present to which the rule that nighttime is absorbed in treachery does no
apply.

In addition, the presence of evident premeditation is likewise borne out by the record. For
the victims were told at the start, when they were taken captives, that they had done
something wrong, that they were the ones who stabbed and killed one Pabling, and that
for this reason they were to go with the group
Not only that; the victims were then taken to a spot where they were ordered to dig their
graves.

The assailants were previously armed with deadly weapons, and their assault was a
concerted and group action. From the time of apprehension of the victims, About 10
o'clock in the evening, to the time Antonio Maravilla lost consciousness, about 1 o'clock
early the following morning, is sufficient time for the offenders to meditate and reflect on
the consequences of their act.

In the present case, we find the facts and circumstances obtaining sufficient to support the
trial
court's finding of the attendance of evident premeditation.

#44.a

PEOPLE OF THE PHILIPPINES, plaintiff-appellee,


vs.
SALVADOR LOMERIO, defendant-appellant.
G.R. No. 129074 (February 28, 2000)

I. FACTS:
On May 23, 1993, Vilma Bunagan, together with her eldest son Roberto, went to
her parents' house at Tuazon St., Marikina, Metro Manila to bring her two-year-old son
who was sick. She left behind in their Antipolo house her other five children, namely,
LEONILA, Marvie, Lotis, Marichu and Edmar, who were the ages 10, 8, 7,6 and 1,
respectively. Worried that something bad could happen to her children while unattended
by an adult companion, Vilma asked her youngest brother, Salvador Lomerio
(SALVADOR), to fetch the children from Antipolo and bring them to Marikina.

https://lookaside.fbsbx.com/file/Book%20I_Consolidated%20C…hqsTJHSGnedWZ0OSOpxZmrtUVtu6RtpxVMMpL7Kt2IHuRsCvg 22/10/2018, 9W25 PM


Page 50 of 90
SALVADOR arrived at the house in Antipolo at about 11:00 p.m. SALVADOR
stayed in the sala smoking cigarettes while LEONILA and her brother and sisters were
lying down. Later, SALVADOR took off his clothes and went near LEONILA.
SALVADOR then got LEONILA's hands and pinned her down. After taking off her
shorts and panty, SALVADOR placed himself on top of LEONILA and then forcibly
inserted his organ in hers for a long time. LEONILA was hurting. All she could do was
cry. SALVADOR went back to the sala and slept.

Again, at about 12:00 midnight of the same date, Salvador went back to
LEONILA and raped her for the second time. SALVADOR threatened LEONILA that he
would kill all of them if she would report the rapes to anybody. Marvie was likewise
raped by SALVADOR in the early morning of the following day. As if nothing untoward
happened, Salvador instructed the children to dress up so they could go to Tuazon in
Marikina and join their mother.

II. ISSUE
Whether the aggravating circumstance of nighttime and abuse of confidence are
present in this case?

III. HELD
No. The elements of nocturnity as an aggravating circumstance are: (a) when it
facilitated the commission of the crime; or (b) when especially sought by the offender to
insure the commission of the crime or for the purpose of impunity, or (c) when the
offender took advantage thereof also for purposes of impunity. There are two tests for
nocturnity as an aggravating circumstance: the objective test, under which nocturnity is
aggravating because it facilitates the commission of the offense; and the subjective test,
under which nocturnity is aggravating because it was purposely sought by the offender.
These two tests should be applied in the alternative. In this case, the subjective test is not
passed because there is no showing that SALVADOR purposely sought the cover of
nighttime. The mere fact that the rape was committed at nighttime with nothing more
does not make nocturnity in this particular case an aggravating circumstance.

As to the aggravating circumstance of abuse of confidence, it is essential to show


that confidence between the parties must be immediate and personal, such as would give
the accused some advantage or make it easier for him to commit the criminal act. The
confidence must be a means of facilitating the commission of the crime, the culprit taking
advantage of the offended parties' belief that the former would not abuse said confidence.
In this case, the bare allegation that the victim's mother asked SALVADOR to fetch her
children from Antipolo to Marikina does not prove that she reposed such confidence in
SALVADOR that he could have used to his advantage in committing the crime.

https://lookaside.fbsbx.com/file/Book%20I_Consolidated%20C…hqsTJHSGnedWZ0OSOpxZmrtUVtu6RtpxVMMpL7Kt2IHuRsCvg 22/10/2018, 9W25 PM


Page 51 of 90
#44.c
THE PEOPLE OF THE PHILIPPINES, plaintiff-appellee,
vs.
FLORO RODIL defendant-appellant.
MAKASIAR, J.:
G.R. No. L-35156
November 20, 1981

FACTS:
Accused Floro Rodil was found guilty for the death of Lt. Guillermo Masana of the
Philippine Constabulary. The accused, armed with a double-bladed dagger, with evident
premeditation and treachery, and with intent to kill, did, attack and stab PC Lt. Guillermo
Masana while the latter was in the performance of his official duties, which directly
caused his death.

ISSUE:
Whether or not the crime of murder can be complexed with assault upon agent of
authority.

https://lookaside.fbsbx.com/file/Book%20I_Consolidated%20C…hqsTJHSGnedWZ0OSOpxZmrtUVtu6RtpxVMMpL7Kt2IHuRsCvg 22/10/2018, 9W25 PM


Page 52 of 90
HELD:
Yes. The Solicitor General claims the crime committed was murder because "it was
established by the prosecution that during the stabbing incident, appellant suddenly and
without giving the victim a chance to defend himself, stabbed the latter several times with
a dagger, inflicting. The suddenness of the attack does not by itself suffice to support a
finding of treachery , the record failed to show that the accused made any preparation to
kill his victim so as to insure the commission of the crime. Clearly, therefore, the
impelling motive for the attack by appellant on his victim was the latter's performance of
official duty, which the former resented. This kind of evidence does not clearly show the
presence of treachery in the commission of the crime.

The aggravating circumstance of disregard of rank should be appreciated because it is


obvious that the victim. Identified himself as a PC officer to the accused who is merely a
member of the Anti-Smuggling Unit and therefore inferior both in rank and social status
to the victim. If the accused herein were charged with the complex crime of murder with
assault against an agent of a person in authority, and not merely murder, then the
aggravating circumstance of disregard of rank or contempt of or insult to public authority
cannot be appreciated as aggravating because either circumstance is inherent in the
charge of assault against a person in authority or an agent of a person in authority.

But in the case at bar, the appellant is accused of murder only. Consequently, either
aggravating circumstance should be considered in the imposition of the penalty.
Appellant guilty of HOMICIDE AGGRAVATED BY CONTEMPT FOR OR INSULT
TO A PUBLIC AUTHORITY OR DISREGARD OF THE RESPECT DUE THE
OFFENDED PARTY ON ACCOUNT OF HIS RANK.

#45
People vs Castillo

Facts: This is an automatic review of the decision of the RTC of Paranaque in Criminal
Case finding appelants Elizabeth Castillo and Evangeline Padayhag guilty of Qualified
Kidnapping and Serious Illegal detention and sentencing them to death. The information
charging Castillo, Padayhag and Imelda Winceslao with the crime of Kidnapping.

Issue: Whether or not the lower court is correct in holding Padayhag guilty as co-
principal by conspiracy in this case.

Held: No. To hold an accused guilty as co-principal by conspiracy, there must be


sufficient and unbroken chain of events that directly and definitely links the accused to
the commission of the crime without any space for baseless supposition or frenzied
theories to filter through; Conspiracy is established by the presence of two factors which
must concur: 1. Singularity of intent and (2.) unity of execution of an unlawful objective.
The two must concur.
​Performance of act contributes to the goal of another is not enough. The act must
be motivated by the same unlawful intent. Neither joint nor simultaneous action is per se
sufficient indicium of conspiracy, unless proved to have motivated by common design.
​Padayhag’s act of fetching Rocky is not conclusive proof of her complicity with
Castillo’s plan, a plan Padayhag did not know.

https://lookaside.fbsbx.com/file/Book%20I_Consolidated%20C…hqsTJHSGnedWZ0OSOpxZmrtUVtu6RtpxVMMpL7Kt2IHuRsCvg 22/10/2018, 9W25 PM


Page 53 of 90
#46
People v. Tan
G.R. No. 132324. September 28, 1999

Facts:

At a distance of about twenty (20) meters, Ramon Nueca saw Magalino Olos, (who was
also his brother-in-law), walking along the road going to Gatbo. At that time, appellant
Jose Tan who was then sixteen (16) years old, was also walking in front of Olos.
Appellant Norlito Tan, (brother of appellant Jose Tan), who was holding an eight-inch
knife known as gatab, suddenly emerged from the grassy portion at the right side of the
road where the grasses [were] higher than a person. Appellant Norlito Tan stabbed Olos
three (3) times. Olos was hit twice at the upper portion of his back and once at his
abdomen. Thereafter, appellant Jose Tan threw a stone at Olos, hitting him at his neck.
Olos was brought by an ambulance to the Bicol Regional Hospital in Naga City where he
later died.

Issues:

(1) Whether the claim of self-defense by Tan will prosper?


(2) Whether the crime was attended by treachery?

Ruling/Ratio:

https://lookaside.fbsbx.com/file/Book%20I_Consolidated%20C…hqsTJHSGnedWZ0OSOpxZmrtUVtu6RtpxVMMpL7Kt2IHuRsCvg 22/10/2018, 9W25 PM


Page 54 of 90
1. No. When the accused invoke self-defense, the burden of proof is shifted to them
to prove that the killing was justified and that they incurred no criminal liability
therefor. They must rely on the strength of their own evidence and not on the
weakness of that of the prosecution, for even if the latter is weak, it could not be
disbelieved after their open admission of responsibility for the killing. In the
present case, it is incumbent upon Appellant Norly Tan to prove self-
defense. Thus, he must prove that there was unlawful aggression on the part of the
victim, that the means employed to prevent it were reasonable, and that there was
lack of sufficient provocation o his part. However, he failed to discharge this
burden.

Yes. The essence of treachery is the sudden and unexpected attack, without the slightest
provocation on the part of the person attacked.[23] Treachery is present when the
offender commits any of the crimes against persons, employing means, methods or forms
in the execution thereof, which tend directly and especially to insure its execution,
without risk arising from the defense which the offended party might make.[24] In the
case at bar, the attack on Magdalino Olos was treacherous, because he was caught off
guard and was therefore unable to defend himself, as testified to by the prosecution
witnesses and as indicated by the wounds inflicted on him.

#48
In the matter of the petition for habeas corpus, Juan Ponce Enrile vs. Salazar

G.R. No. 92163, June 5, 1990

FACTS:

In the afternoon of February 27, 1990, Senate Minority Floor Leader Juan Ponce Enrile
was arrested by law enforcement officers led by Director Alfredo Lim of the National
Bureau of Investigation on the strength of a warrant issued by Hon. Jaime Salazar. The
warrant had issued on an information charging Senator Enrile with the crime of rebellion
with murder and multiple frustrated murder allegedly committed during the period of his
failed coup attempt. No bail was recommended.

On February 28, 1990, Senator Enrile, filed a petition for habeas corpus alleging that he
was being held to answer for a criminal offense which does not exist in the statute books
and that he was arrested and detained on the strength of a warrant issued without the
judge first having personally determined the existence of probable cause. Enrile invoked
the landmark case of People vs. Hernandez where it was ruled that rebellion cannot be
complexed with common crimes such as murder.

ISSUES:

AJ. WON the Hernandez ruling should be abandoned.


AK. WON Judge Salazar was able to personally determine probable cause.

HELD:

https://lookaside.fbsbx.com/file/Book%20I_Consolidated%20C…hqsTJHSGnedWZ0OSOpxZmrtUVtu6RtpxVMMpL7Kt2IHuRsCvg 22/10/2018, 9W25 PM


Page 55 of 90
2. NO. The Hernandez ruling is still good law, its substantive and logical bases have
withstood all subsequent challenges and no new ones were presented persuasive enough
to warrant a complete reversal. This view is reinforced by the fact that then President
Cory Aquino, exercising her powers under the 1986 Freedom Constitution, saw fit to
repeal, among others, Presidential Decree No. 942 of the Marcos regime which precisely
sought to nullify or neutralize Hernandez by enacting a new provision (Art. 142-A) into
the Revised Penal Code to the effect that "(w)hen by reason, or on the occasion, of any of
the crimes penalized in this Chapter (Chapter I of Title 3, which includes rebellion), acts
which constitute offenses upon which graver penalties are imposed by law are committed,
the penalty for the most serious offense in its maximum period shall be imposed upon the
offender." In thus acting, the President in effect reinstated Hernandez as binding doctrine
with the effect of law. Hence, in the instant case, Enrile may only be charged with simple
rebellion.

YES. For the Court, there is nothing irregular on the conduct of Judge Salazar who only
took one hour and twenty minutes to issue the warrant. Accordingly, it is sufficient that
the judge follows established procedure by personally evaluating the report and the
supporting documents submitted by the prosecutor. Merely because Judge Salazar had
what some might consider only a relatively brief period within which to comply with his
duty, gives no reason to assume that he had not, or could not have, so complied; nor does
that single circumstance suffice to overcome the legal presumption that official duty has
been regularly performed.

#49
G.R. No. 109266 ​December 2, 1993

MIRIAM DEFENSOR SANTIAGO, petitioner,


vs.

HON. JUSTICE FRANCIS GARCHITORENA, SANDIGANBAYAN


(First Division) and PEOPLE OF THE PHILIPPINES, respondents.

Facts:

Petitioner was charged in the Sandiganbayan with violation of Sec. 3(e) of RA


3019, Anti-Graft and Corrupt Practices Act, allegedly committed by her favoring
"unqualified" aliens with the benefits of the Alien Legalization Program.
Petitioner filed this case to enjoin Sandiganbayan from proceeding with the case,
on the ground that it was intended solely to harass her as she was then a
presidential candidate. After her petition was dismissed, she then filed a motion
for inhibition of Presiding Justice Garchitorena.

[A lot of procedural issues and controversies were discussed, but for the
purpose of limiting this digest to Criminal Law 1, the author did not include
it.]

Petitioner next claims that the Amended Informations did not charge any offense
punishable under Section 3 (e) of R.A. No. 3019 because the official acts
complained of therein were authorized under Executive Order No. 324 and that
the Board of Commissioners of the Bureau of Investigation adopted the policy of
approving applications for legalization of spouses and unmarried, minor children
of "qualified aliens" even though they had arrived in the Philippines after

https://lookaside.fbsbx.com/file/Book%20I_Consolidated%20C…hqsTJHSGnedWZ0OSOpxZmrtUVtu6RtpxVMMpL7Kt2IHuRsCvg 22/10/2018, 9W25 PM


Page 56 of 90
December 31, 1983. She concludes that the Sandiganbayan erred in not granting
her motion to quash the informations.

In a motion to quash, the accused admits hypothetically the allegations of fact in


the Information. Therefore, petitioner admitted hypothetically in her motion that:
1) she was a public officer; 2) she approved the application for legalization of the
stay of aliens, who arrived in the Philippines after January 1, 1984; 3) those aliens
were disqualified; 4) she was cognizant of such fact; and 5) she acted in evident
bad faith and manifest partiality in the execution of her official functions; thereby
constituting the elements of the offense defined in Sec. 3(e) of RA 3019.

It bears noting that the public prosecutors filed a total of 32 Informations


against the petitioner for the violation of such law.

Issue:

How is the violation of Sec. 3(e) of RA 3019 committed?

Held:

There are two ways of violating Section 3 (e) of R.A. No. 3019. These are: (a)
by causing undue injury to any party, including the Government; and (b) by
giving any private party any unwarranted benefit, advantage or preference.

Issue #2:

Whether or not the filing of 32 Amended Informations against petitioner


was proper.

Held: NO.

Only one crime was committed in petitioner’s case, and hence, there should only
be one Information to be filed against her.

The 32 Amended Informations charge what is known as delito continuado or

"continued crime" and sometimes referred to as "continuous crime." A delito


continuado consists of several crimes but in reality there is only one crime in
the mind of the perpetrator. See full text for the discussion and examples of
delito continuado as discussed by SC.

In the case at bench, the original information charged petitioner with performing
a single criminal act — that of her approving the application for legalization of
aliens not qualified under the law to enjoy such privilege. The original
information also averred that the criminal act : (i) committed by petitioner was
in violation of a law

— Executive Order No. 324 dated April 13, 1988, (ii) caused an undue injury to
one offended party, the Government, and (iii) was done on a single day, i.e., on

https://lookaside.fbsbx.com/file/Book%20I_Consolidated%20C…hqsTJHSGnedWZ0OSOpxZmrtUVtu6RtpxVMMpL7Kt2IHuRsCvg 22/10/2018, 9W25 PM


Page 57 of 90
or about October 17, 1988. The 32 Amended Informations reproduced in
verbatim the allegation of the original information, except that instead of the
word "aliens" in the original information each amended information states the
name of the individual whose stay was legalized.

The 32 Amended Informations aver that the offenses were committed on the
same period of time, i.e., on or about October 17, 1988. The strong probability
even exists that the approval of the application or the legalization of the stay of
the 32 aliens was done by a single stroke of the pen, as when the approval was
embodied in the same document. Likewise, the public prosecutors manifested at
the hearing the motion for a bill of particulars that the Government suffered a
single harm or injury.

SC ordered the Ombudsman to consolidated the 32 Amended Informations into one


information charging only one offense.

#51
THE PEOPLE OF THE PHILIPPINES, plaintiff-appellee, vs. MARIO
TABACO, accused-appellant.

Facts:
In the evening of March 22, 1987, the 17th PC stationed at Aparri, Cagayan, under then
Lt. James Andres Melad, sponsored a cock derby, under the name of Jose Ting, at the
Octagon Cockpit Arena located at Aparri, Cagayan.
This being so, peace officers in uniform with long firearms were assigned as guards to
maintain peace and order at the cockpit arena namely: (1) Sgt. Benito Raquepo; (2) CIS
Roque P. Datugan, both from the 117th PC and (3) Pat. Andles Semana, INP, Aparri,
Cagayan. Accused Mario Tabaco who was in civilian clothes claims to have been also
assigned by his Commanding Officer of 117th PC, to verify the presence of NPAs and
assist in the protection of VIPs in the cockpit arena, bringing with him his M-14 issued
firearm.
Other peace officers who came to participate were: (1) Policeman Mariano Retreta of
INP, Buguey, Cagayan, who arrived with the deceased Jorge Siriban and Licerio
Antiporda, Jr., Licerio Antiporda II; (2) Sgt. Rogelio Ferrer of 117th PC Company; (3)
Policeman Romeo Regunton (deceased) who was also armed, arrived in company with
the deceased Ex-Mayor Arreola; (4) Fireman Rogelio Guimmayen, INP Buguey; (5) Pat.
Barba; and (6) CIC PC Paragas.
At about nine (9) o'clock in the evening of same date, the group of the late Mayor Jorge
Arreola of Buguey, Cagayan, arrived at the cockpit arena. His companions were (1)
Antonio Villasin; (2) Rosario Peneyra; (3) victim Lorclo Pita, Jr. and/or five (5) of them
including the Mayor. They occupied and were (4th row) north western part cockpit-gate.
Others seated with the Mayor were: (1) the late Capt. Oscar Tabulog; (2) the late Pat.
Romeo Regunton, who was at the back of the mayor; (3) the late Felicito Rigunan. The
accused CIC Tabaco was seated on the arm of the bench situated at the lower portion of
the arena about more than three (3) meters away, (infront and a little bit in the west), from
the place where the late Mayor and his group were seated (at the 4th row of seats upper
portion). During the ocular inspection conducted, the Court noticed the distance to be
more than three (3) meters, and/or probably 4-5 meters.
At about ten(10) o'clock 1987, while the accused Mario Tabaco was seated as described
above, he suddenly without warning or provocation, shot the late mayor Jorge Arreola,

https://lookaside.fbsbx.com/file/Book%20I_Consolidated%20C…hqsTJHSGnedWZ0OSOpxZmrtUVtu6RtpxVMMpL7Kt2IHuRsCvg 22/10/2018, 9W25 PM


Page 58 of 90
with his M-14 rifle, followed by several successive burst of gunfire, resulting in the
shooting to death of the late Mayor Arreola, Capt. Oscar Tabulog, Felicito Rigunan and
Pat. Romeo Regunton, although the latter managed to run passing through the western
gate near the gaffers cage but was chased by accused Tabaco. Regunton was later found
dead inside the canteen of Mrs. Amparo Go inside the Octagon cockpit arena.
Pat. Mariano Retreta of INP Buguey, who was then at the Co's canteen, saw the accused
going out rushing from the cockpit arena, at a distance of one meter. Pat. Retreta is a
relative and neighbor of the accused Tabaco in Buguey, Cagayan. He tried to pacify
Tabaco telling him 'what is that happened again Mario.' Meanwhile, Sgt. Benito Raquepo
of 117th PC, and one of those assigned to maintain peace and order at the Octagon
cockpit arena, who was at the canteen taking snacks, heard five (5) successive gun reports
coming from inside the cockpit arena. In a little while, he saw the accused Tabaco
coming from inside the cockpit arena. Raquepo advised Tabaco 'Mario relax ka
lang' 'Mario keep calm.' They stood face to face holding their rifles and when Tabaco
pointed his gun towards Sgt. Raquepo, Pat. Retreta grappled for the possession of the gun
to disarm Tabaco, and in the process, the gun went off hitting Sgt. Raquepo and also the
late Jorge Siriban who happened to be near Raquepo. Siriban died on the spot while
Raquepo survived his wounds on his legs due to adequate medical treatment.
Another circumstance which maybe considered as adverse against the accused, is the fact
that he was really arrested and not that he voluntarily surrendered as appearing in the INP
Lallo Police Blotter, as testified to by Pat. Melin Bautista.
Furthermore, it appears that the same accused Mario Tabaco, has still a pending case for
murder before Branch 6, of this Court.
The accused was found guilty in all four (4) murder charges and the penalty of reclusion
perpetua should have been imposed on him in all four (4) murder charges, the trial court
imposed the penalty of reclusion perpetua for all four murder charges.

Issue:
Whether or not the court was correct in imposing one (1) reclusion perpetua for all four
(4) murder charges?

Held:
The trial court was in error in imposing only a single penalty of reclusion perpetua for all
four murder cases. The trial court holding that a complex crime was committed since "the
evidence shows that the four (4) victims were FELLED by one single shot/burst of fire
]
and/or successive automatic gun fires, meaning continuous does not hold water.
The death of each of the victims who were killed by accused-appellant and the physical
injuries inflicted upon each of the two other persons injured were not caused by the
performance by the accused of one simple act.Although it is true that several successive
shots were fired by the accused in a short space of time, yet the factor which must be
taken into consideration is that, to each death caused or physical injuries inflicted upon
the victims, corresponds a distinct and separate shot fired by the accused, who thus made
himself criminally liable for as many offenses as those resulting from every single act that
produced the same. Although apparently he perpetrated a series of offenses successively
in a matter of seconds, yet each person killed and each person injured by him became the
victim, respectively, of a separate crime of homicide or frustrated homicide. Except for
the fact that five crimes of homicide and two cases of frustrated homicide were
committed successively during the tragic incident, legally speaking there is nothing that
would connect one of them with its companion offenses.

https://lookaside.fbsbx.com/file/Book%20I_Consolidated%20C…hqsTJHSGnedWZ0OSOpxZmrtUVtu6RtpxVMMpL7Kt2IHuRsCvg 22/10/2018, 9W25 PM


Page 59 of 90
#52
MIGUEL CRISTOBAL vs. ALEJO LABRADOR, ET AL.
G.R. No. L-47941 December 7, 1940
FACTS:
On March 15, 1930, the CFI of Rizal found Teofilo C. Santos guilty of the crime of estafa
and sentenced him to six months of arresto mayor and the accesories provided by law, to
return to the offended parties, Toribio Alarcon and Emilio Raymundo the amounts P375
and P125, respectively, with subsidiary imprisonment in the case of insolvency, and to
pay the costs. Accordingly, he was confined in the provincial jail of Pasig, Rizal, from
March 14, 1932 to August 18, 1932 and paid the corresponding costs of trial. As to his
civil liability, the same was condoned by the complaints. Notwithstanding his conviction,
Santos continued to be a registered elector in the municipality of Malabon, Rizal, and
was, for the period comprised between 1934 and 1937, seated as the municipal president.
On August 22, 1938, Commonwealth Act No. 357, otherwise known as the Election
Code, was approved by the national Assembly, section 94, paragraph (b) of which
disqualifies the respondent from voting for having been "declared by final judgment
guilty of any crime against the property." In view of this provision, Santos applied to the
President for an absolute pardon which was granted restoring his "full civil and political
rights, except that with respect to the right to hold public office or employment, he will
be eligible for appointment only to positions which are clerical or manual in nature and
involving no money or property responsibility."
Petitioner filed a petition for the exclusion of the name of Teofilo C. Santos from the list
of voters in precinct No. 11 of Malabon, Rizal, on the ground that the latter is disqualified
under paragraph (b) of section 94 of CA No. 357. After hearing, the court below ruled in
favor of Santos citing the pardon extended to the latter has the effect of excluding him
from disqualification. Cristobal has filed the present petition for certiorari in which he
impugns the decision of the court below on the following grounds: (a) the pardoning
power of the Chief Executive does not apply to legislative prohibitions; (b) the pardoning
power here would amount to an unlawful exercise by the Chief Executive of a legislative
function, and (c) the respondent having served his sentence and all the accesory penalties
imposed by law, there was nothing to pardon.
ISSUE: Whether Santos should be excluded from the list of voters. – NO.
RULING:
There are two limitations upon the exercise of the constitutional prerogative by the Chief
Executive, namely: (a) that the power be exercised after convictions; and (b) that such
power does not extend to cases of impeachment. An absolute pardon not only blots out
the crime committed, but removes all disabilities resulting from the convictions. In the
present case, the disability is the result of conviction without which there would no basis
for disqualification from voting. Imprisonment is not the only punishment which the law
imposes upon those who violate its command. There are accessory and resultant
disabilities, and the pardoning power likewise extends to such disabilities. When granted
after the term of imprisonment has expired, absolute pardon removes all that is left of the
consequences of conviction. In the present case, while the pardon extended to respondent
Santos is conditional in the sense that "he will be eligible for appointment only to
positions which are clerical or manual in nature involving no money or property
responsibility," it is absolute insofar as it "restores the respondent to full civil and
political rights." While there are cases in the United States which hold that the pardoning
power does not restore the privilege of voting, this is because, as stated by the learned

https://lookaside.fbsbx.com/file/Book%20I_Consolidated%20C…hqsTJHSGnedWZ0OSOpxZmrtUVtu6RtpxVMMpL7Kt2IHuRsCvg 22/10/2018, 9W25 PM


Page 60 of 90
judge below, in the United States the right of suffrage is a matter exclusively in the hands
of the State and not in the hands of the Federal Government. Upon the other hand, the
suggestion that the disqualification imposed in paragraph (b) of section 94 of C.A. No.
357, does not fall within the purview of the pardoning power of the Chief Executive,
would lead to the impairment of the pardoning power of the Chief Executive, not
contemplated in the Constitution, and would be no way of restoring the political privilege
in a case of this nature except through legislative action.

https://lookaside.fbsbx.com/file/Book%20I_Consolidated%20C…hqsTJHSGnedWZ0OSOpxZmrtUVtu6RtpxVMMpL7Kt2IHuRsCvg 22/10/2018, 9W25 PM


Page 61 of 90
#54
Case: RICARDO PARULAN, petitioner, vs. DIRECTOR OF PRISONS, respondent
G.R. No. L-28519
February 17, 1968
FACTS:
Parulan was serving his sentence of 20 years in Muntinglupa Rizal, and was transferred
to the military barracks Fort Bonifacio in 1964. There, he was able to escape, but was
later recaptured in Manila. He was prosecuted and found guilty under Art 157 (evasion of
service sentence) by Court of First Instance of Manila. Later, petitioner files a petition
for a writ of habeas corpus praying that the Director be ordered to release him
immediately and without delay from unlawful and illegal confinement. He claims that
the sentence of conviction imposed upon him for the crime of evasion of service of
sentence (penalized under Art 157 RPC) was rendered by a court without jurisdiction
over his person and of the offense with which he was charged.
ISSUE:

Whether the Court of First Instance of Manila has jurisdiction to try and decide the case
and to impose the sentence upon the petitioner for the charge of evasion of service of
sentence?

RULING:
Yes, it did. There are crimes which are called transitory or continuing offenses
because some acts material and essential to the crime occur in one province and some in
another, in which case, the rule is settled that the court of either province where any of
the essential ingredients of the crime took place has — jurisdiction to try the case.
​It may not be validly said that after the convict shall have escaped from the place
of his confinement the crime is fully consummated, for, as long as he continues to evade
the service of his sentence, he is deemed to continue committing the crime, and may be
arrested without warrant, at any place where he may be found.

https://lookaside.fbsbx.com/file/Book%20I_Consolidated%20C…hqsTJHSGnedWZ0OSOpxZmrtUVtu6RtpxVMMpL7Kt2IHuRsCvg 22/10/2018, 9W25 PM


Page 62 of 90
#55
G.R. No. 179187 July 14, 2009
PEOPLE OF THE PHILIPPINES, Appellee, vs. RENATO TALUSAN y
PANGANIBAN, Appellant.

FACTS: In the early morning of January 14, 2004, as AAA (6years old) was on her way
to school, appellant, who was sitting by a tree in Las Pinas, pulled her aside and cajoled
her into joining him by telling her that they would go to Jollibee. AAA obliged as she
knew appellant to be a fellow attendee of Sunday Bible classes. Appellant brought AAA,
however, to a house in Imus, Cavite occupied by one Eljoy Salonga and two unidentified
individuals to whom he introduced her as his daughter. AAA was thereafter under
appellants control and custody for eight days during which he abused her by inserting his
finger inside her vagina on a daily basis before breakfast, despite her resistance.
Appellant was charged with kidnapping with rape and he pleaded guilty. The lower court
thereupon conducted a searching inquiry into the voluntariness of appellants plea, and
despite repeated questions and just as repeated answers showing that appellant
understood his plea and its consequences, the trial court still ordered the prosecution to,
as it did, present evidence. RTC convicted him and imposed death. The CA modified the
penalty to Reclusion perpetua.
ISSUE: Whether or not the trial court erred in convicting the appellant on the basis of an
improvident plea of guilt as it failed to judiciously follow the guidelines set forth in
People v. Pastor?
HELD: NO. There is thus no hard and fast rule as to how a judge may conduct a
searching inquiry. As long as the voluntary intent of the accused and his full
comprehension of the consequences of his plea are ascertained, as was done in the
present case, the accused plea of guilt is sustained.
In the present case, even without the plea of guilt of appellant, the evidence presented by
the prosecution supports his guilt beyond reasonable doubt of the special complex crime
of kidnapping with rape.
A review of the evidence for the prosecution shows that the actual confinement, restraint
and rape of AAA were proven. AAA, a minor whose testimony is given full faith and
credit, youth and immaturity being generally badges of truth and sincerity. AAAs
stepfather BBB testified on her disappearance for eight days and the measures he took in
order to recover her. And the initial medicolegal report conducted for inquest purposes
shows that AAA suffered deep fresh lacerations in her hymen which are compatible with
recent loss of virginity.
NOTE: In Pastor, the Court, holding that there is no definite and concrete rule as to how
a trial judge must conduct a searching inquiry, nevertheless came up with the following
guidelines:
1. Ascertain from the accused himself (a) how he was brought into the custody of
the law; (b) whether he had the assistance of a competent counsel during
the custodial and preliminary investigations; and (c) under what
conditions he was detained and interrogated during the investigations.
This is intended to rule out the possibility that the accused has been
coerced or placed under a state of duress either by actual threats of
physical harm coming from malevolent quarters or simply because of
the judge's intimidating robes.

2. Ask the defense counsel a series of questions as to whether he had conferred


with, and completely explained to, the accused the meaning and
consequences of a plea of guilty.

https://lookaside.fbsbx.com/file/Book%20I_Consolidated%20C…hqsTJHSGnedWZ0OSOpxZmrtUVtu6RtpxVMMpL7Kt2IHuRsCvg 22/10/2018, 9W25 PM


Page 63 of 90
3. Elicit information about the personality profile of the accused, such as his age,
socioeconomic status, and educational background, which may serve as
a trustworthy index of his capacity to give a free and informed plea of
guilty.

4. Inform the accused the exact length of imprisonment or nature of the penalty
under the law and the certainty that he will serve such sentence. For not
infrequently, an accused pleads guilty in the hope of a lenient treatment
or upon bad advice or because of promises of the authorities or parties
of a lighter penalty should he admit guilt or express remorse. It is the
duty of the judge to ensure that the accused does not labor under these
mistaken impressions because a plea of guilty carries with it not only the
admission of authorship of the crime proper but also of the aggravating
circumstances attending it, that increase punishment.

5. Inquire if the accused knows the crime with which he is charged and fully
explain to him the elements of the crime which is the basis of his
indictment. Failure of the court to do so would constitute a violation of
his fundamental right to be informed of the precise nature of the
accusation against him and a denial of his right to due process.

6. All questions posed to the accused should be in a language known and


understood by the latter.

7. The trial judge must satisfy himself that the accused, in pleading guilty, is truly
guilty. The accused must be required to narrate the tragedy or reenact
the crime or furnish its missing details.

#56
MAJOR GENERAL CARLOS F. GARCIA, AFP (RET.), Petitioner, vs.THE
EXECUTIVE SECRETARY, representing the OFFICE OF THE PRESIDENT;
THE SECRETARY OF NATIONAL DEFENSE VOLTAIRE T. GAZMIN; THE
CHIEF OF STAFF, ARMED FORCES OF THE PHILIPPINES, GEN. EDUARDO
SL. OBAN, JR., and LT. GEN. GAUDENCIO S. PANGILINAN, AFP (RET.),
DIRECTOR, BUREAU OF CORRECTIONS, Respondents.

FACTS: A Charge Sheet dated October 27, 2004 was filed with the Special General Court

https://lookaside.fbsbx.com/file/Book%20I_Consolidated%20C…hqsTJHSGnedWZ0OSOpxZmrtUVtu6RtpxVMMpL7Kt2IHuRsCvg 22/10/2018, 9W25 PM


Page 64 of 90
Martial NR 2 presided by Maj. Gen. Emmanuel R. Teodosio, AFP, (Ret.), enumerating
the following violations allegedly committed by petitioner: · CHARGE 1: VIOLATION
OF THE 96TH ARTICLE OF WAR (CONDUCT UNBECOMING AN OFFICER AND
GENTLEMAN) for failure to disclose all his existing assets in his SALN for the years
2002 and 2003 and that he violated his oath as a military officer by acquiring and holding
the status of an immigrant/permanent residence of the United States of America ·
CHARGE II: VIOLATION OF THE 97TH ARTICLE OF WAR (CONDUCT
PREJUDICIAL TO GOOD ORDER AND MILITARY DISCIPLINE) by making
untruthful statements under oath of his true assets in his SALN for the years 2002 and
2003. Petitioner, upon arraignment on November 16, 2004, pleaded not guilty on all the
charges. The Office of the Chief of Staff directed the transfer of confinement of petitioner
from his quarters at Camp General Emilio Aguinaldo to the ISAFP Detention Center. On
the same day, petitioner, having reached the age of fifty-six (56), compulsorily retired
from military service after availing of the provisions of Presidential Decree No. 1650,
amending Sections 3 and 5 of P.D. 1638, which establishes a system of retirement for
military personnel of the Armed Forces of the Philippines. After trial, at the Special
General Court Martial No. 2, found him guilty. The Office of the President, or the
President as Commander-in-Chief of the AFP and acting as the Confirming Authority
under the Articles of War, confirmed the sentence imposed by the Court Martial against
petitioner.
Issue:
Whether or not the jurisdiction of the general court martial ceased ipso facto upon the
retirement of the petitioner, for which reason the office of the president acted without
jurisdiction in issuing the confirmation of sentence and petitioner’s arrest and
confinement pursuant thereto is illegal, thus warranting the writ of habeas corpus?

HELD:
NO. It is indisputable that petitioner was an officer in the active service of the AFP in
March 2003 and 2004, when the alleged violations were committed. The charges were
filed on October 27, 2004 and he was arraigned on November 16, 2004. Clearly, from the
time the violations were committed until the time petitioner was arraigned, the General
Court Martial had jurisdiction over the case. Well-settled is the rule that jurisdiction once
acquired is not lost upon the instance of the parties but continues until the case is
terminated. Therefore, petitioner's retirement on November 18, 2004 did not divest the
General Court Martial of its jurisdiction.
Having established the jurisdiction of the General Court Martial over the case and the
person of the petitioner, the President, as Commander-in-Chief, therefore acquired the
jurisdiction to confirm petitioner's sentence as mandated under Article 47 of the Articles
of War The General Court Martial is a court within the strictest sense of the word and acts
as a criminal court. On that premise, certain provisions of the Revised Penal Code,
insofar as those that are not provided in the Articles of War and the Manual for Courts-
Martial, can be supplementary.

https://lookaside.fbsbx.com/file/Book%20I_Consolidated%20C…hqsTJHSGnedWZ0OSOpxZmrtUVtu6RtpxVMMpL7Kt2IHuRsCvg 22/10/2018, 9W25 PM


Page 65 of 90
#58
G.R. No. L-33254 & G.R. No. L-33253 January 20, 1978
THE PEOPLE OF THE PHILIPPINES, plaintiff-appellee, vs. LICERIO P.
SENDAYDIEGO, JUAN SAMSON and ANASTACIO
QUIRIMIT, defendants. JUAN SAMSON and defendant-appellant.
PROVINCE OF PANGASINAN, offended party-appellee, vs. HEIRS OF LICERIO P.
SENDAYDIEGO, defendants-appellants. *

Facts:

https://lookaside.fbsbx.com/file/Book%20I_Consolidated%20C…hqsTJHSGnedWZ0OSOpxZmrtUVtu6RtpxVMMpL7Kt2IHuRsCvg 22/10/2018, 9W25 PM


Page 66 of 90
• In these three cases of malversation through falsification, the prosecution's theory is that
in 1969 Licerio P. Sendaydiego, the provincial treasurer of Pangasinan, in conspiracy
with Juan Samson y Galvan, an employee of a lumber and hardware store in Dagupan
City, and with Anastacio Quirimit, the provincial auditor, as an accomplice, used six
(6) forged provincial vouchers in order to embezzle from the road and bridge fund the
total sum of P57,048.23.
• The provincial voucher in these cases has several parts. In the upper part with the legend
"ARTICLE OR SERVICE" the nature of the obligation incurred is indicated. That part
is supposed to be signed by two officials of the provincial engineer's office and by the
governor's representative.
• The middle part of the voucher contains five numbered printed paragraphs. Paragraph 1
is a certificate to be signed by the creditor. It is stated therein that the creditor vouches
that the expenses "were actually and necessarily incurred".
• In the instant cases paragraph 1 was not signed presumably because it is not relevant to
the purchase of materials for public works projects. Paragraph 2 is a certification that
the expenses are correct and have been lawfully incurred. It is signed by the provincial
engineer. Paragraph 3 contains these words: "Approved for pre-audit and payment,
appropriations and funds being available therefore." This is signed by the provincial
treasurer. Paragraph 4 is a certification, to wit: “I certify that this voucher has been
pre-audited and same may be paid in the amount of sixteen thought seven hundred
twenty-seven and 52/100 (P16,727.52) in cash or in check, provided there is sufficient
fund cover the payment.” This is signed by the auditor. Paragraph 5 is a certification
signed by the provincial treasurer that the account mentioned in the provincial
engineer's certification "was paid in the amount and on the date shown below and is
chargeable as shown in the summary hereof. ... ." It may be noted that the provincial
treasurer signs two part of the voucher. Following paragraph 5, and as referred to
therein, is the receipt of the signed by the creditor. The receipt reads (it was signed
according to the prosecution by Juan Samson, a point which is disputed by him):
“Received this 31st day of March, 1969, from L P. Sendaydiego, Province of
Pangasinan the sum of seven hundred twenty-seven pesos & 52/100 (16,727.52) in
full payment of the above stated account, which I hereby certify to be correct. Paid by
Check No. ................................. CARRIED CONSTR. SUPPLY CO. By: (Sgd.)
JUAN SAMSON.”
• According to the prosecution, Samson also signed on the left margin of the six vouchers
below the stamped words: "Presented to Prov. Treasurer. By Juan Samson."
• The six (6) forged provincial vouchers, with their respective supporting papers, were
hand-carried by Samson. Afterwards, Samson asked Donato Rosete the assistant
provincial treasurer, to initialled the voucher After Rosete had initialled the vouchers,
Samson went to the provincial treasurer's office where the amounts covered by the
voucher were paid by Sendaydiego to him in cash (instead of by check) as
representative of the Carried Construction Supply Co. The signature of Sendaydiego
and Quirimit, the auditor, on the said six vouchers are admittedly authentic.
Sendaydiego signed the vouchers ahead of Rosete, his assistant. Sendaydiego's
defense is that he signed the vouchers in the honest belief that the signatures therein of
the provincial office concerned were genuine because the voucher had been pre-
audited and approved by the auditor.
• Lower court – acquitted the auditor, Quirimit and found Sendaydiego and Samnson
guilty of malversation through falsification of public or official documents.
• Sendaydiego died on October 5, 1976. His appeal as to his criminal liability was
dismissed. Death extinguished his criminal liability remained. Sendaydiego's appeal
will be resolved only for the purpose of showing his criminal liability which is the
basis of the civil liability for which his estate would be liable for which his estate
would be liable.

https://lookaside.fbsbx.com/file/Book%20I_Consolidated%20C…hqsTJHSGnedWZ0OSOpxZmrtUVtu6RtpxVMMpL7Kt2IHuRsCvg 22/10/2018, 9W25 PM


Page 67 of 90
Held:
• Sendaydiego's death had rendered moot the issue as to the propriety of the imposition
of reclusion perpetua. And, as will be shown later, reclusion perpetua cannot be
imposed in these cases because the crimes committed were not complex.
• We are convinced that his criminal liability was established beyond reasonable doubt
and, therefore, the civil liability fo his estate for the amounts malversed was duly
substantial.
• The trial court assumed that three complex crimes of malversation through falsification
of public documents were committed in this case. That assumption is wrong.
• The crimes committed in these three cases are not complex. Separate crimes of
falsification and malversation were committed. These are not cases where the
execution of a single act constitutes two grave or less grave felonies or where the
falsification was used as a means to commit malversation.
• In the six vouchers the falsification was used to conceal the malversation. It is settled
that if the falsification was resorted to for the purpose of hiding the malversation, the
falsification and malversation are separate offenses.
• In the instant cases, the provincial , as the custodian than of the money forming part of
the road and bridge could have malversed or misappropriated it without falsifiying
any voucher. The falsification was used as a device to prevent detection of the
malversation.
• The falsifications cannot be regarded as constituting one continuing offense impelled by
a single criminal impulse. Each falsification of a voucher constitutes one crime. The
falsification of six vouchers constitutes six separate or distinct offenses.

https://lookaside.fbsbx.com/file/Book%20I_Consolidated%20C…hqsTJHSGnedWZ0OSOpxZmrtUVtu6RtpxVMMpL7Kt2IHuRsCvg 22/10/2018, 9W25 PM


Page 68 of 90
#59
REODICA V CA, 292 SCRA 87, GR NO. 125066, JULY 8, 1998

APPLICABLE PROVISION: Art 365. Imprudence and negligence;

FACTS: One evening, petitioner Isabelita Reodica was driving a van along Doña
Soledad Avenue, Better Living Subdivision, Parañaque, Metro Manila. Allegedly because
of her recklessness, her van hit the car of complainant Norberto Bonsol. As a result,
complainant sustained physical injuries, while the damage to his car amounted to
P8,542.00.

Three days after the incident, the complainant filed an Affidavit of Complaint against
petitioner with the Fiscal's Office. An information was filed before the RTC of Makati
charging petitioner with "Reckless Imprudence Resulting in Damage to Property with
Slight Physical Injury RTC of Makati, rendered a decision convicting petitioner of the
"quasi offense of reckless imprudence resulting in damage to property with slight
physical injuries," and sentencing her: to suffer imprisonment of six (6) months of arresto
mayor, and to pay the complainant, Norberto Bonsol y Atienza, the sum of Thirteen
Thousand Five Hundred Forty-Two (P13,542), Philippine Currency, without subsidiary
impairment in case of insolvency; and to pay the costs.

ISSUES:


1. Whether the penalty imposed on petitioner is correct.

2. Whether the quasi offenses of reckless imprudence resulting in damage to property in


the amount of P8,542.00 and reckless imprudence resulting in slight physical injuries are
light felonies.

3. Whether the rule on complex crimes under Article 48 of the Revised Penal Code
applies to the quasi offenses in question.

RULING:

ISSUE1: NO. The Court agreed with both petitioner and the OSG that the penalty of six
months of arresto mayor imposed by the trial court and affirmed by respondent Court of
Appeals is incorrect. However, we cannot subscribe to their submission that the penalty
of arresto menor in its maximum period is the proper penalty.

Art. 48. Penalty for complex crimes. — When a single act constitutes two or more grave

https://lookaside.fbsbx.com/file/Book%20I_Consolidated%20C…hqsTJHSGnedWZ0OSOpxZmrtUVtu6RtpxVMMpL7Kt2IHuRsCvg 22/10/2018, 9W25 PM


Page 69 of 90
or less grave felonies, or when an offense is necessary a means for committing the other,
the penalty for the most serious crime shall be imposed, the same to be applied in its
maximum period.

According to Art 365, the penalty for reckless imprudence resulting in slight physical
injuries, a light felony, is arresto menor in its maximum period, with a duration of 21 to
30 days. If the offense of slight physical injuries is, however, committed deliberately or
with malice, it is penalized with arresto menor under Article 266 of the Revised Penal
Code, with a duration of 1 day to 30 days. Plainly, the penalty then under Article 266 may
be either lower than or equal to the penalty prescribed under the first paragraph of Article
365. This being the case, the exception in the sixth paragraph of Article 365 applies.
Hence, the proper penalty for reckless imprudence resulting in slight physical injuries is
public censure, this being the penalty next lower in degree to arresto menor.

As to reckless imprudence resulting in damage to property in the amount of P8,542.00,


the third paragraph of Article 365, which provides for the penalty of fine, does not apply
since the reckless imprudence in this case did not result in damage to property only. What
applies is the first paragraph of Article 365, which provides for arresto mayor in its
minimum and medium periods (1 month and 1 day to 4 months) for an act committed
through reckless imprudence which, had it been intentional, would have constituted a less
grave felony. Note that if the damage to the extent of P8,542.00 were caused deliberately,
the crime would have been malicious mischief under Article 329 of the Revised Penal
Code, and the penalty would then be arresto mayor in its medium and maximum periods
(2 months and 1 day to 6 months which is higher than that prescribed in the first
paragraph of Article 365). If the penalty under Article 329 were equal to or lower than
that provided for in the first paragraph, then the sixth paragraph of Article 365 would
apply, i.e., the penalty next lower in degree, which is arresto menor in its maximum
period to arresto mayor in its minimum period or imprisonment from 21 days to 2
months. Accordingly, the imposable penalty for reckless imprudence resulting in damage
to property to the extent of P8,542.00 would be arresto mayor in its minimum and
medium periods, which could be anywhere from a minimum of 1 month and 1 day to a
maximum of 4 months, at the discretion of the court, since the fifth paragraph of Article
365 provides that in the imposition of the penalties therein provided "the courts shall
exercise their sound discretion without regard to the rules prescribed in article 64."

ISSUE2: In reckless imprudence resulting in slight physical injuries is punishable by


public censure only. Article 9, paragraph 3, of the Revised Penal Code defines light
felonies as infractions of law carrying the penalty of arresto menor or a fine not
exceeding P200.00, or both. Since public censure is classified under Article 25 of the
Code as a light penalty, and is considered under the graduated scale provided in Article
71 of the same Code as a penalty lower than arresto menor, it follows that the offense of
reckless imprudence resulting in slight physical injuries is a light felony.

On the other hand, reckless imprudence also resulting in damage to property is, as
earlier discussed, penalized with arresto mayor in its minimum and medium periods.
Since arresto mayor is a correctional penalty under Article 25 of the Revised Penal Code,
the quasi offense in question is a less grave felony — not a light felony as claimed by

petitioner.

ISSUE NO. 3. NO. Clearly, if a reckless, imprudent or negligent act results in two or
more grave or less grave felonies, a complex crime is committed. However, in Lontok
v.Gorgonio, 27 this Court declared that where one of the resulting offenses in criminal
negligence constitutes a light felony, there is no complex crime, thus:

Applying article 48, it follows that if one offense is light, there is no complex crime. The
resulting offenses may be treated as separate or the light felony may be absorbed by the

https://lookaside.fbsbx.com/file/Book%20I_Consolidated%20C…hqsTJHSGnedWZ0OSOpxZmrtUVtu6RtpxVMMpL7Kt2IHuRsCvg 22/10/2018, 9W25 PM


Page 70 of 90
grave felony. Thus, the light felonies of damage to property and slight physical injuries,
both resulting from a single act of imprudence, do not constitute a complex crime. They
cannot be charged in one information. They are separate offenses subject to distinct
penalties (People vs. Turla, 50 Phil. 1001; See People vs. Estipona, 70 Phil.

513).

Hence, the trial court erred in considering the following felonies as a complex crime: the
less grave felony of reckless imprudence resulting in damage to property in the amount of
P8,542.00 and the light felony of reckless imprudence resulting in physical injuries.
Applying article 48, it follows that if one offense is light, there is no complex crime. The
resulting offenses may be treated as separate or the light felony may be absorbed by the
grave felony. Thus, the light felonies of damage to property and slight physical injuries,
both resulting from a single act of imprudence, do not constitute a complex crime. They
cannot be charged in one information. They are separate offenses subject to distinct
penalties (People vs. Turla, 50 Phil. 1001; See People vs. Estipona, 70 Phil. 513). Hence,
the trial court erred in considering the following felonies as a complex crime: the less
grave felony of reckless imprudence resulting in damage to property in the amount of
P8,542.00 and the light felony of reckless imprudence resulting in physical injuries.

https://lookaside.fbsbx.com/file/Book%20I_Consolidated%20C…hqsTJHSGnedWZ0OSOpxZmrtUVtu6RtpxVMMpL7Kt2IHuRsCvg 22/10/2018, 9W25 PM


Page 71 of 90
#60
Colinares
vs.
People of the Philippines
Facts:
Arnel Colinares was charged and found guilty beyond reasonable doubt of frustrated
homicide by the RTC of Camarines Sur. He was sentenced to suffer imprisonment from
two years and four months of prison correccional, as minimum, to six years and one day
of prison mayor, as maximum. Since the maximum probationable imprisonment under
the law was only up to six years, Arnel did not qualify for probation. On appeal by
Colinares, the Court of Appeals sustained the RTC’s decision. Unsatisfied with the Court
of Appeal’s decision, petitioner then appealed to the Supreme Court and took the position
that he should be entitled to apply for probation in case the Court metes out a new penalty
on him that makes his offense probationable, which was strongly opposed by the Solicitor
General reiterating that under the Probation Law, no application for probation can be
entertained once the accused has perfected his appeal from the judgment of conviction.
The Supreme Court, however, found that Colinares is guilty of attempted homicide and
not of frustrated homicide.
Issue:
Whether or not Arnel Colinares may still apply for probation on remand of the case to the
trial court
Ruling:
Yes, The Supreme Court ruled that Colinares may apply for probation upon remand of his
case to the RTC. Ordinarily, an accused would no longer be entitled to apply for
probation, he having appealed from the judgment of the RTC convicting him for
frustrated homicide. But in this case the Supreme Court ruled to set aside the judgment of
the RTC and found him only liable for attempted homicide, if the Supreme Court follows
the established rule that no accused can apply for probation on appeal, the accused would
suffer from the erroneous judgment of the RTC with no fault of his own, therefore
defying fairness and equity.

https://lookaside.fbsbx.com/file/Book%20I_Consolidated%20C…hqsTJHSGnedWZ0OSOpxZmrtUVtu6RtpxVMMpL7Kt2IHuRsCvg 22/10/2018, 9W25 PM


Page 72 of 90
#61
FRANCISCO vs. CA
G.R. No. 108747, April 6, 1995

Topic: Probation

FACTS:
Pablo Francisco was accused of multiple grave oral defamation by his employees.
The MTC sentenced him of prision correccional in its minimum period in each crime
committed on each date of each case. Francisco then elevated the case to the RTC in
which they sentenced him only of eight straight months for appreciating mitigating
circumstances.
Francisco failed to make an appeal on the RTC’s decision making it final. The
MTC issued a warrant of arrest, but before Francisco was to be arrested, he filed an
application for probation which the MTC denied. He went to the Court of Appeals on
certiorari which was also denied.

ISSUE:
Whether or not Francisco is still qualified to avail of probation.

RULING:
Negative. Petitioner is no longer eligible for probation.

First, Francisco violated Sec.4 of the Probation Law in which no application for
probation shall be entertained after the judgment is final. Probation is a mere privilege,
not a right. Its benefits cannot extend to those not expressly included.
Second, the penalties imposed by the MTC were already probationable. Hence,
there was no need to appeal if only to reduce the penalties to within the probationable
period. Multiple prison terms imposed against an accused found guilty of several offenses
in one decision are not, and should not be, added up. And, the sum of the multiple prison
terms imposed against an applicant should not be determinative of his eligibility for, nay

https://lookaside.fbsbx.com/file/Book%20I_Consolidated%20C…hqsTJHSGnedWZ0OSOpxZmrtUVtu6RtpxVMMpL7Kt2IHuRsCvg 22/10/2018, 9W25 PM


Page 73 of 90
his disqualification from, probation. Petitioner should have immediately filed an
application for probation as he was already qualified after being convicted by the MTC.
Consequently, Francisco lost his right to probation when he appealed the MTC decision
to the RTC. The law considers appeal and probation mutually exclusive remedies.
Third, Francisco’s appeal to the RTC was not for reducing his penalties but for his
assertion of his innocence. The Probation Law prevents opportunism when petitioners
apply for probation when their appeal was dismissed.
Lastly, the application for probation was filed way beyond the period allowed by
law. The application for probation was filed only after a warrant of had been issued and
almost two (2) months after his receipt of the Decision of the RTC. This is a significant
fact which militates against the instant petition.

#62
People of the Philippines vs. Astorga
G.R. No. 110097. December 22, 1997

Topic: Motive; Spanish text controlling

Facts:
Arnulfo Astorga, accused, told Yvonne Traya, eight years old, to go with him to buy
candy. She did not answer and the accused immediately grabbed her and held her hand.
They went to Maco Elementary School and strolled on the school grounds. When nobody
was at the Luponlupon bridge, the accused took the victim to the highway leading to
Tagum, Davao. At that time, Yvonne pleaded with the accused that she really wanted to
go home to Binuangan, but accused ignored her pleas and continued walking her toward
the wrong direction. Later on, the group of Witness Arnel Fabila spotted them. Appellant
Astorga carried the victim and ran, but Fabilas group chased and caught up with them.
Astorga was then charged with kidnapping and the trial court convicted him. The accused
contends that the prosecution failed to prove any motive on why the accused should
kidnap Yvonne. He also contends that the prosecution failed to prove one essential
element of kidnapping -- the fact of detention or the deprivation of liberty.

Issue:
(1) Whether or not the prosecution needs to prove motive on the part of the accused?
(2) Whether or not accused should be convicted of kidnapping?

Held:
(1) No. Motive is not an element of the crime. Furthermore, motive becomes material
only when the evidence is circumstantial or inconclusive, and there is some doubt on
whether a crime has been committed or whether the accused has committed it. Indeed,
motive is totally irrelevant when ample direct evidence sustains the culpability of the
accused beyond reasonable doubt. In this case, the identity of appellant is not in question.
He himself admitted having taken Yvonne to Maco Central Elementary School.

https://lookaside.fbsbx.com/file/Book%20I_Consolidated%20C…hqsTJHSGnedWZ0OSOpxZmrtUVtu6RtpxVMMpL7Kt2IHuRsCvg 22/10/2018, 9W25 PM


Page 74 of 90
(2) No. The Spanish version of Article 267 of the Revised Penal Code uses the terms
lockup (encerrar) rather than kidnap (secuestrar or raptar). Lockup is included in the
broader term of detention, which refers not only to the placing of a person in an enclosure
which he cannot leave, but also to any other deprivation of liberty which does not
necessarily involve locking up. Likewise, the Revised Penal Code was originally
approved and enacted in Spanish. Consequently, the Spanish text is controlling in cases
of conflict with the English version, as provided in Section 15 of the Revised
Administrative Code. A review of the events as narrated by the prosecution witnesses
ineluctably shows the absence of locking up. Since there was no lock up, appellant cannot
be convicted of kidnapping. Rather, the felony committed in this case is grave coercion
under Article 286.

#65
PEOPLE OF THE PHILIPPINES vs. IRVIN TADULAN y EPAN
G.R. No. 117407 | April 15, 1997
PADILLA, J.

DOCTRINE / RULING:
Alibi; It has been held time and again that for alibi to prosper as a defense the accused
must show that he was so far away that he could not have been physically present at the
place of the crime, or its immediate vicinity at the time of its commission (People vs.
Tasurra, 192 SCRA 266). In this case, however, it is not so situated, for according to him
he was at the plant of the Republic Asahi Glass Corporation in Barangay Pinagbuhatan,
Pasig, —which is but a few kilometers from Barangay Sumilang of the same municipality
where the crime was committed.

Rape; Accused-appellant tries to discredit the victim’s testimony by questioning her


behavior after she was allegedly raped. The court ruled that it is not proper to judge the
actions of children who have undergone traumatic experience by the norms of behavior
expected under the circumstances from mature people. The range of emotion shown by
rape victims is yet to be captured even by the calculus. It is thus unrealistic to expect
uniform reactions from rape victims.It should be borne in mind, in this connection, that
the victim was only a naive nine (9) year old child when the crime was committed on her.
She considered the accused as a friend, almost like a relative, as in fact she called him
“Tito Loloy.”

As correctly observed by the Solicitor General: “(A)s regards the acts imputed to Estela,
the delay of seven (7) days from the date of her knowledge of the rape incident on 4 April
1992 in reporting to the authorities the rape of her daughter is excusable. At that time, she
was not yet certain of the steps she would take considering the delicate nature of the
problem they were facing” (citing People v. Danguilan, 218 SCRA 98; People v. Joaquin,
Jr., 225 SCRA 179). Besides, we have ruled that a delay in prosecuting the rape is not
indicative of fabricated charges.

Pardon; It is clear to the mind of this Court that the complainant has not expressly
pardoned the said accused. Besides, there are authorities holding that pardon must be
granted not only by the parents of an offended minor but also by the minor herself in
order to be effective as an express pardon under Art. 344 of the Revised Penal Code.
Thus, in the case of People vs. Lacson, Jr., (C.A.) 55 O.G. 9460, we find the following
words: ‘Neither must we be understood as supporting the view that the parents alone can
extend a valid pardon. Far from it, for we, too are of the belief that the pardon by the

https://lookaside.fbsbx.com/file/Book%20I_Consolidated%20C…hqsTJHSGnedWZ0OSOpxZmrtUVtu6RtpxVMMpL7Kt2IHuRsCvg 22/10/2018, 9W25 PM


Page 75 of 90
parents, standing alone, is inefficacious.’ It was also held in another case, that ‘The
express pardon of a person guilty of attempted abduction of a minor, granted by the
latter’s parents, is not sufficient to remove criminal responsibility, but must be
accompanied by the express pardon of the girl herself.’ (U.S. vs. Luna, 1 Phil. 360).

FACTS:
Complainant Estela Santos owns a house at No. 6 Dr. Garcia St., in Barangay Sumilang,
Pasig, she resides with her common-law husband and their minor daughter, Maristel
Cruz. Behind the said house, complainant also owns a three-door apartment building, one
unit of which was rented and occupied by accused Irvin Tadulan, his wife Adefa Tadulan
and their three children name Dianne, Angie and Bochoy who were aged 10, 9 and 5,
respectively. In 1992 complainant’s daughter, Maristel Cruz was about nine (9) year old
and was in grade school. She often played with the accused’s children in the vicinity of
their house and the apartment building.

That on or about the 2nd day of April, 1992 in the Municipality of Pasig, Metro Manila,
Philippines and within the jurisdiction of this Honorable Court, armed with a knife, with
lewd design and by means of force, threats and intimidation, did then and there willfully,
unlawfully and feloniously have sexual intercourse with one Maristel Cruz, a minor, nine
(9) years old, without her consent and against her will.

Estela Santos immediately informed the wife of Irvin Tadulan that her husband has raped
her daughter. She further informed Adefa Tadulan that she would not take action against
the latter’s husband if they would vacate the apartment unit right away. Adefa Tadulan
later on met with Estela Santos and told her that she had driven away Irvin Tadulan, but
requested that she and her children be allowed to stay until Saturday, April 11, 1992.
Estela Santos thereafter noted, however, that Irvin Tadulan was still coming home to the
apartment unit every night despite the promise of his wife that she herself would call the
police should he ever come back to the place.

ISSUE:
Whether or not the court erred in disregarding the defense of pardon and alibi of the
accused?

https://lookaside.fbsbx.com/file/Book%20I_Consolidated%20C…hqsTJHSGnedWZ0OSOpxZmrtUVtu6RtpxVMMpL7Kt2IHuRsCvg 22/10/2018, 9W25 PM


Page 76 of 90
#68
G.R. NO. 140231
PCGG vs HON. ANIANO A. DESIERTO

Facts:

Then President Fidel V. Ramos issued Administrative Order No. 13 creating the
Presidential Ad Hoc Fact-Finding Committee on Behest Loans (Committee) which was
tasked to inventory all behest loans, determine the parties involved and recommend
whatever appropriate actions to be pursued thereby. President Ramos issued
Memorandum Order No. 61 to include inventory and review of all non-performing loans,
whether behest or non-behest. Among the accounts referred to the Committee's Technical
Working Group (TWG) were the loan transactions between NOCOSII and PNB.The loan
was classified as behest because of NOCOSIIs insufficient capital and inadequate
collaterals. Petitioner filed with the Office of the Ombudsman the criminal complaint
against respondents. Petitioner alleges that respondents violated the following provisions
of Section 3 (e) and (g) of R.A. No. 3019. GIO Diaz-Salcedo recommended the dismissal
of the case on the ground of insufficiency of evidence or lack of probable cause.
Ombudsman Desierto approved the recommendation

Issue: Whether the Ombudsman committed grave abuse of discretion in ruling that: (a)
the offense leveled against respondents has prescribed.

Ruling:

Yes. Respondent Ombudsman committed grave abuse of discretion in dismissing the


subject complaint on the ground of prescription

Respondents members of the PNB Board of Directors and Officers of NOCOSII are
charged with violation of R.A. No. 3019, a special law. Amending said law, Section 4,
Batas Pambansa Blg. 195,[11] increased the prescriptive period from ten to fifteen years.
The applicable law in the computation of the prescriptive period is Section 2 of Act No.
3326. Prescription shall begin to run from the day of the commission of the violation of
the law, and if the same not be known at the time, from the discovery thereof and the
institution of judicial proceedings for its investigation and punishment. The prescription
shall be interrupted when proceedings are instituted against the guilty person, and shall
begin to run again if the proceedings are dismissed for reasons not constituting jeopardy.
It was well-nigh impossible for the State, the aggrieved party, to have known the
violations of R.A. No. 3019 at the time the questioned transactions were made because,
as alleged, the public officials concerned connived or conspired with the beneficiaries of
the loans. Thus the prescriptive period for the offenses should be computed from the
discovery of the commission thereof and not from the day of such commission. The
counting of the prescriptive period commenced from the date of discovery of the offense
in 1992 after an exhaustive investigation by the Presidential Ad Hoc Committee on
Behest Loans.
Prescription is interrupted when proceedings are instituted against the guilty person.
Records show that the act complained of was discovered in 1992. The complaint was

https://lookaside.fbsbx.com/file/Book%20I_Consolidated%20C…hqsTJHSGnedWZ0OSOpxZmrtUVtu6RtpxVMMpL7Kt2IHuRsCvg 22/10/2018, 9W25 PM


Page 77 of 90
filed with the Office of the Ombudsman on April 5, 1995,[17] or within three (3) years
from the time of discovery. Thus, the filing of the complaint was well within the
prescriptive period of 15 years.
Nonetheless, evaluation of the records of this case reveals that the loans acquired by
NOCOSII are actually foreign loans from Midland Bank Ltd. of London. There were no
direct loans released by PNB but merely credit accommodations to guaranty the loans
from Midland Bank. The herein assailed Orders being supported by substantial evidence,
there is no basis for the Court to exercise its supervisory powers over the ruling of the
Ombudsman. As long as substantial evidence supports the Ombudsmans ruling, that
decision will not be overturned.

#69
SOCIAL SECURITY SYSTEM v DEPARTMENT OF JUSTICE, JOSE V.
MARTEL, OLGA S. MARTEL, and SYSTEMS AND ENCODING
CORPORATION
G.R. No. 158131 August 8, 2007
Topic: Novation as mode of extinguishing criminal liability

Facts:
In 1998, petitioner filed with the Pasay City Prosecutor’s Office a complaint
against respondent Martels and their five co-accused for SENCOR’s non-payment of
contributions. To pay this amount, respondent Martels offered to assign to petitioner a
parcel of land in Tagaytay City covered by Transfer Certificate of Title No. 26340 issued
under respondent Martels’ name. Petitioner accepted the offer "subject to the condition
that respondent Martels will settle their obligation either by way of dacion en pago or
through cash settlement within a reasonable. Thus, petitioner withdrew its complaint
from the Pasay City Prosecutor’s Office but reserved its right to revive the same "in the
event that no settlement is arrived at.

Issue:
Whether or not the novation can extinguish criminal liability.

Held:
No. It may be observed in this regard that novation is not one of the means
recognized by the Penal Code whereby criminal liability can be extinguished; hence, the
role of novation may only be to either prevent the rise of criminal liability or to cast
doubt on the true nature of the original basic transaction, whether or not it was such that
its breach would not give rise to penal responsibility, as when money loaned is made to
appear as a deposit, or other similar disguise is resorted to.
The facts of this case negate the application of novation. In the first place, there is,
between SENCOR and petitioner, no original contract that can be replaced by a new
contract changing the object or principal condition of the original contract, substituting
the person of the debtor, or subrogating a third person in the rights of the creditor. The
original relationship between SENCOR and petitioner is defined by law – RA 1161, as
amended – which requires employers like SENCOR to make periodic contributions to
petitioner under pain of criminal prosecution. Unless Congress enacts a law further
amending RA 1161 to give employers a chance to settle their overdue contributions to
prevent prosecution, no amount of agreements between petitioner and SENCOR
(represented by respondent Martels) can change the nature of their relationship and the
consequence of SENCOR’s non-payment of contributions.

https://lookaside.fbsbx.com/file/Book%20I_Consolidated%20C…hqsTJHSGnedWZ0OSOpxZmrtUVtu6RtpxVMMpL7Kt2IHuRsCvg 22/10/2018, 9W25 PM


Page 78 of 90
#71
G.R. No. 100285
August 13, 1992
PEOPLE OF THE PHILIPPINES
vs.
NAPOLEON DUQUE

FACTS:
​In January 1986, at the house of Napoleon Duque in Calamba, Laguna, he met
with the complainants, Glicerio Teodoro, Agustin Ulat, Ernesto Maunahan, Norma
Francisco, Elmo Alcaraz and Marcelino Desepida. He made the complainants believe that
he was authorized and licensed as a recruiter for workers for placement in abroad. He
received money from the complainants which he said he will use to acquire pertinent
documents for the complainants’ employment. He also made them believe that they will
be able to leave in two months. Despite repeated demands of the complainants for a
receipt for the monies Duque received, he did not issue any to the complainants. This
prompted the complainants to file a case of illegal recruitment against Duque before the
Philippine Overseas Employment Agency (POEA).
​Duque denied all allegations, but said that his house was used as a meeting place
for a certain Delfin and one Engr. Acopado who allegedly were the persons who had
promised complainants, work abroad.
​The trial court ruled in favour of the complainants. Hence, Duque came to the
Supreme Court stating that the period to file a complaint for his crime has prescribed.

ISSUE:
Whether or not Duque’s offense of illegal recruitment has prescribed.

HELD:
​No. The Labor Code does not contain any provisions on the mode of computation
of the three-year prescriptive period it established. But Act No. 3326, as amended,
entitled "An Act to Establish Periods of Prescription for Violations Penalized by Special
Acts and Municipal Ordinances and to Provide When Prescription Shall Begin to Run"
states that: “Prescription shall begin to run from the day of the commission of the
violation of the law, and if the same be not known at the time, from the discovery thereof
and institution of judicial proceedings for its investigation and punishment.”
The provided provision on Act No.3326 shows that there are two (2) rules for
determining the beginning of the prescriptive period: (a) on the day of the commission of
the violation, if such commission be known; and (b) if the commission of the violation
was not known at the time, then from discovery thereof and institution of judicial
proceedings for investigation and punishment.
Duque contends that the prescriptive period in the case at bar commenced from
the time money in consideration of promises for overseas employment was parted with
by complainants. Duque thus contends that the prescriptive period began to run sometime
in January 1986. The information was, however, filed by the Assistant Provincial
Prosecutor of Laguna on 22 May 1990, i.e., more than four (4) years later. Duque
concludes that the offense of illegal recruitment had accordingly prescribed by May 1990.

https://lookaside.fbsbx.com/file/Book%20I_Consolidated%20C…hqsTJHSGnedWZ0OSOpxZmrtUVtu6RtpxVMMpL7Kt2IHuRsCvg 22/10/2018, 9W25 PM


Page 79 of 90
The lack of necessary permit or authority, while certainly known to appellant
Duque back in January 1986, was not known to private complainants at that time. Indeed,
private complainants discovered that appellant did not possess such authority or permit
only when they went to the offices of the POEA for the purpose of filing a claim for
return of the money they had delivered to appellant Duque. Since good faith is always
presumed, the complainants were entitled to assume the appellant Duque was acting in
good faith when he presented himself as a recruiter for overseas placement. Even if it be
assumed arguendo that ordinary prudence required that a person seeking overseas
employment ought to check the authority or status of persons pretending to be authorized
or to speak for a recruitment or placement agency, the offended parties' failure to do so
did not start the running of the prescriptive period. In the nature of things, acts made
criminal by special laws are frequently not immoral or obviously criminal in themselves;
for this reason, the applicable statute requires that if the violation of the special law is not
known at the time, then prescription begins to run only from the discovery thereof, i.e.,
discovery of the unlawful nature of the constitutive act or acts.
We believe and so hold that the applicable prescriptive period in the case at bar
began to run from the time the recruitment activities of appellant Duque were ascertained
by the complainants and by the POEA to have been carried out without any license or
authority from the government. The discovery by the complainants and by the POEA
was, as a practical matter, simultaneous in character and occurred sometime in December
1989 when the complainants went to the POEA with the complaint for recovery of the
placement fees and expenses they had paid to appellant Duque, and the POEA, acting
upon that complaint, discovered and informed the private complainants that Duque had
operated as a recruiter without the essential government license or authority. Accordingly,
the offense of illegal recruitment had not prescribed when the complaint was filed with
the Provincial Prosecutor's Office in April 1990 and when the information was filed in
court in May 1990.
It is relevant to note that the same result would be reached by giving supplemental
effect to provisions of the Revised Penal Code in the application of Article 290 of the
Labor Code. 8 Article 91 of the Revised Penal Code reads as follows:
Art. 91. Computation of the prescription of offenses. — The period of
prescription shall commence to run from the day on which the crime is discovered by the
offended party, the authorities, or their agents, and shall be interrupted by the filing of the
complaint or information, and shall commence to run again when such proceedings
terminate without the accused being convicted or acquitted, or are unjustifiably stopped
for any reason not imputable to him.
The term of prescription shall not run when the offender is absent from the
Philippine Archipelago.

#72
G.R. No. 177763 July 3, 2013
PEOPLE OF THE PHILIPPINES, Plaintiff-Appellee, vs.
1
GARY VERGARA y ORIEL and JOSEPH INOCENCIO y PAULINO, Accused-
Appellants.
Doctrine:
There is Treachery when the offender commits any of the crimes against the person,
employing means, methods, or forms in the execution thereof which tend directly and
specially to insure its execution, without risk to himself arising from the defense which
the offended party might

https://lookaside.fbsbx.com/file/Book%20I_Consolidated%20C…hqsTJHSGnedWZ0OSOpxZmrtUVtu6RtpxVMMpL7Kt2IHuRsCvg 22/10/2018, 9W25 PM


Page 80 of 90
Facts:
2
Before this Court is an appeal of the March 30, 2007 Decision of the Court of Appeals
4
affirming with modification the Decision of the RTC Branch 116, Pasay City entitled
People of the Philippines v. Gary Vergara and Joseph Inocencio " finding accused-
appellants Gary Vergara and Joseph Inocencio guilty beyond reasonable doubt of murder
as principal and accomplice, respectively.
The prosecution established that at around midnight of February 10, 2001, accused-
appellants were causing a ruckus on Libertad-Colayco Streets, Pasay City by throwing
water bottles at passers-by. At around 2:00 a.m., the victim, Miguelito Alfante, who was
seemingly drunk, walked down the street. Vergara approached Alfante and told him:
"Pare, mukhang high na high ka." Alfante retorted: "Anong pakialam mo?" At this
juncture, Vergara threw his arm around Alfante’s shoulder, received a knife from
Inocencio, and suddenly stabbed Alfante. Vergara then said "Taga rito ako." Thereafter,
Vergara and Inocencio ran from the scene but were pursued by several witnesses. Alfante,
meanwhile, was brought to the Pasay City General Hospital where he died. The autopsy
report conducted on the cadaver of the victim revealed that Alfante sustained eight stab
wounds.
In his defense, Vergara denied the version of the prosecution. After evaluating the
respective evidence of the contending parties, on December 29, 2001, the RTC found
accused-appellants guilty beyond reasonable doubt of the crime of murder as defined
under Article 248 of the Revised Penal Code. As in the Court of Appeals, accused-
appellants challenge the court a quo’s finding of guilt beyond reasonable doubt. They
averred that the elements of the crime of murder were not proven. On March 30, 2007,
the Court of Appeals affirmed with modification as to the award of damages the Decision
of the RTC.
Issue:
Whether or not the killing was attended with Treachery thus considered as Murder?
Held:
The death of the victim, Miguelito Alfante, is directly caused by the stab wounds inflicted
by appellant Vergara when he placed his left arm on the shoulder of the victim and
stabbed him repeatedly in his chest and left forearm with a knife handed to him by
appellant Inocencio. This is an overwhelming evidence, and in stark contrast, all
appellant Vergara could offer are denial and self-defense. Anent accused-appellant
Vergara’s claim of self-defense, he must prove all the elements of self-defense, most
important of all is unlawful aggression on the part of the victim. Unlawful aggression
must be proved first in order for self-defense to be successfully pleaded, whether
complete or incomplete. In the present case, the element of unlawful aggression is absent.
We also agree with the RTC and the Court of Appeals that the acts of accused-appellant
Vergara constituted treachery qualifying the crime committed to murder. As we have
previously ruled upon, treachery is present when the offender commits any of the crimes
against persons, employing means, methods, or forms in the execution, which tend
directly and specially to insure its execution, without risk to the offender arising from the
defense which the offended party might make.
Here, accused-appellant Vergara after exchanging words with the victim, threw his arm
around the victim’s shoulder and proceeded to stab him. The victim was totally unaware
of the evil that would befall him. The number and severity of the wounds received by the
victim indicated that he was rendered immobile and without any real opportunity to
defend himself other than feebly raising his arm to ward off the attack. The Court
sustained the trial court and the Court of Appeals in finding that the qualifying
circumstance of treachery is present in the commission of the crime.
Article 248 of the Revised Penal Code, as amended by Republic Act No. 7659, provides
for the penalty of reclusion perpetua to death for the crime of murder. Though there was
an appreciation of voluntary surrender as a mitigating circumstance, following the
Indeterminate Sentence Law, the RTC, as affirmed by the Court of Appeals, properly

https://lookaside.fbsbx.com/file/Book%20I_Consolidated%20C…hqsTJHSGnedWZ0OSOpxZmrtUVtu6RtpxVMMpL7Kt2IHuRsCvg 22/10/2018, 9W25 PM


Page 81 of 90
imposed the penalty of reclusion perpetua, pursuant to Article 63, paragraph 2, of the
Revised Penal Code.

#73
People vs. Talaro
FACTS:

On 24 April 1994, Raymundo Zamora, nephew of Gregorio Talaro (one of the accused),
saw Francisca Talaro (his Uncle Gregorio’s wife), Lolito Aquino, Renato Atong Ramos,
and Norberto Jun Adviento, who are all accused in the present case, conversing among
themselves under a santol tree in front of his (Zamora's) house. He went near the group to
find out what they were talking about and learned that his Aunt Francisca was transacting
with the other three accused for the killing of Atty. Melvin Alipio. He was merely a meter
away from the group so he heard the group's conversation. He learned that Francisca
Talaro would give the three accused-appellants an advance payment of P30,000.00 and
then another P30,000.00 after Atty. Melvin Alipio is killed, with said last payment to be
delivered in Barangay (Brgy.) Bactad. The three accused-appellants then nodded their
heads in agreement. Then, in the morning of 26 April 1994, Rodolfo Duzon, one of the
accused, was at the parking area in the poblacion of Urdaneta waiting for passengers,
when accused-appellant Renato Ramos hired him as tricycle driver in a trip to Laoac,
Pangasinan for Php 200.00. According to him, Ramos alighted the tricycle upon arriving
in said place, particularly at Guardian Angel Hospital, which is owned by spouses Atty.
Melvin Alipion and Dr. Lina Alipio. Five minutes later, Duzon heard three (3) gunshots
coming from the west. Then, he saw Ramos coming toward him, being chased by another
man, who was later identified as Rene Balanga, helper of the Alipio spouses. Meanwhile,
Eusebio Hidalgo, one of the eyewitnesses, testified that the gunman was among those
persons who sat with him in a bench while waiting outside the Spouses Alipio’s clinic,
where his son was confined. He likewise identified the gunman as Renato Ramos.

The aforesaid accused were charged before the Regional Trial Court (RTC) of Urdaneta,
Pangasinan, with the crime of murder. The RTC found Norberto (Jun) Adviento, Renato
Ramos, and Lolito Aquino guilty beyond reasonable doubt of the crime of Murder and
sentenced them to death. The case was then brought to this Court for automatic review in
view of the penalty of death imposed on accused-appellants. However, in accordance
with the ruling in People v. Mateo,[17] and the amendments made to Sections 3 and 10 of
Rule 122, Section 13 of Rule 124, and Section 3 of Rule 125 of the Revised Rules on
Criminal Procedure, the Court transferred this case to the Court of Appeals (CA) for
intermediate review. Then, the CA affirmed the trial court’s decision, but acquitted Duzon
on reasonable doubt. Undaunted, the accused elevated their case to the Supreme Court for
final disposition.

https://lookaside.fbsbx.com/file/Book%20I_Consolidated%20C…hqsTJHSGnedWZ0OSOpxZmrtUVtu6RtpxVMMpL7Kt2IHuRsCvg 22/10/2018, 9W25 PM


Page 82 of 90
In the Supreme Court level:

AL. Accused Lolito Aquino and Norberto Adviento argued that (1) they should be
acquitted because the prosecution evidence is insufficient to prove they were part
of the conspiracy to kill Atty. Melvin Alipio, and that (2) there are no aggravating
circumstances that would justify the imposition of death penalty; and

AM. Accused Renato Ramos maintains that his identity was never established.

ISSUES:

3. Were Aquino and Adviento part of the conspiracy to kill Atty. Alipio?
4. Was Ramos’ identity as killer established sufficiently by the prosecution?

RULING:

3. YES. In this regard, the Honorable Court held:

“… the existence of a conspiracy has been established by the testimony of


Raymundo Zamora, positively identifying all three accused-appellants as the ones
he saw and heard transacting with Francisca Talaro on April 24, 1994 to kill Atty.
Melvin Alipio for the price of P60,000.00, and pointing to Lolito Aquino as the
one who demanded and received part of the payment after Atty. Alipio had been
killed. The credibility of Raymundo Zamora's testimony is further bolstered by
Lolito Aquino's admission that he and Renato Ramos even conducted surveillance
on the victim a day before Renato Ramos carried out the shooting, and that the
motorcycle used as a getaway vehicle belonged to him. Rodolfo Duzon also
pointed to Renato Ramos as the gunman; he also pointed to Renato Ramos and
Norberto (Jun) Adviento as the ones who threatened to kill him if he talks to
anyone about the shooting. All the proven circumstances point to the conclusion
that accused-appellants acted in concert to assure the success of the execution of
the crime; hence, the existence of a conspiracy is firmly established.”

4. Yes. Thus, the Court held:

“Ramos was properly identified in open court by Raymundo Zamora, as one of the men
he saw and heard transacting with Francisca Talaro for the killing of Atty. Alipio.[30]
Hence, there can be no doubt as to which Renato Ramos is being convicted for the
murder of Atty. Alipio.”

https://lookaside.fbsbx.com/file/Book%20I_Consolidated%20C…hqsTJHSGnedWZ0OSOpxZmrtUVtu6RtpxVMMpL7Kt2IHuRsCvg 22/10/2018, 9W25 PM


Page 83 of 90
#74

DAYAP VS SENDIONG

Article 365 of the Revised Penal Code punishes any person who, by reckless
imprudence, commits any act which, had it been intentional, would constitute a grave
felony, with the penalty of arresto mayor in its maximum period to prision correccional
in its medium period. When such reckless imprudence the use of a motor vehicle,
resulting in the death of a person attended the same article imposes upon the defendant
the penalty of prision correccional in its medium and maximum periods;

Where a reckless, imprudent, or negligent act results in two or more grave or less
grave felonies, a complex crime is committed. Article 48 of the Revised Penal Code
provides that when the single act constitutes two or more grave or less grave felonies, or
when an offense is a necessary means for committing the other, the penalty for the most
serious crime shall be imposed, the same to be applied in its maximum period. Since
Article 48 speaks of felonies, it is applicable to crimes through negligence in view of the
definition of felonies in Article 3 as “acts or omissions punishable by law” committed
either by means of deceit (dolo) or fault (culpa). Thus, the penalty imposable upon
petitioner, were he to be found guilty, is prision correccional in its medium period (2
years, 4 months and 1 day to 4 years) and maximum period (4 years, 2 months and 1 day
to 6 years).

FACTS:
Dayap allegedly drove in a reckless manner a 10-wheeler cargo truck hitting an
automobile driven by Sendiong who was with two female passengers. Such incident
caused the death of Sendiong, less serious physical injuries on the bodies of the two
female passengers and extensive damage to the automobile. Hence, Dayap was charged
with the crime of Reckless Imprudence resulting to Homicide, Less Serious Physical
Injuries, and Damage to Property.

ISSUE:
WON the RTC has jurisdiction to hear a criminal case involving complex crimes such as
reckless imprudence resulting in homicide, less serious physical injuries and damage to
property?

RULING:
NO. When this case was filed on 29 December 2004, Section 32(2) of Batas Pambansa

https://lookaside.fbsbx.com/file/Book%20I_Consolidated%20C…hqsTJHSGnedWZ0OSOpxZmrtUVtu6RtpxVMMpL7Kt2IHuRsCvg 22/10/2018, 9W25 PM


Page 84 of 90
Bilang 129 had already been amended by R.A. No. 7691. R.A. No. 7691 extended the
jurisdiction of the first-level courts over criminal cases to include all offenses punishable
with imprisonment not exceeding six (6) years irrespective of the amount of fine, and
regardless of other imposable accessory or other penalties including those for civil
liability. It explicitly states "that in offenses involving damage to property through
criminal negligence, they shall have exclusive original jurisdiction thereof." It follows
that criminal cases for reckless imprudence punishable with prision correccional in
its medium and maximum periods should fall within the jurisdiction of the MTC
and not the RTC. Clearly, therefore, jurisdiction to hear and try the same pertained to
the MTC and the RTC did not have original jurisdiction over the criminal
case. Consequently, the MTC of Sibulan, Negros Oriental had properly taken cognizance
of the case and the proceedings before it were valid and legal.

https://lookaside.fbsbx.com/file/Book%20I_Consolidated%20C…hqsTJHSGnedWZ0OSOpxZmrtUVtu6RtpxVMMpL7Kt2IHuRsCvg 22/10/2018, 9W25 PM


Page 85 of 90
#76
Safeguard Security Agency, Inc vs Tangco
G.R. No.165732
December 14, 2006

Facts:

One afternoon, Evangeline Tangco (Evangeline) went to Ecology Bank, Katipunan


Branch, Quezon City, to sign the specimen card for the renewal of her time deposit as
advised of the bank's cashier. Evangeline, a duly licensed firearm holder with
corresponding permit to carry, approached security guard Pajarillo (employee of
Safeguard Security Agency, Inc.), who was stationed outside the bank, and pulled out her
firearm from her bag to deposit the same for safekeeping. Suddenly, Pajarillo shot
Evangeline with his service shotgun hitting her in the abdomen, causing her death. He
argued that when Evangeline approached the bank, she was seen pulling a gun from
inside her bag and petitioner Pajarillo who was suddenly beset by fear and perceived the
act as a dangerous threat, shot and killed the deceased out of pure instinct; that the act of
drawing a gun is a threatening act, regardless of whether or not the gun was intended to
be used against petitioner Pajarillo; that the fear that was created in the mind of petitioner
Pajarillo as he saw Evangeline Tangco drawing a gun from her purse was suddenly very
real and the former merely reacted out of pure self-preservation.

Issues :
Whether Pajarillo is guilty of negligence in shooting Evangeline.

Held: Yes.
Considering that unlawful aggression on the part of Evangeline is absent, Pajarillos claim
of self-defense cannot be accepted specially when such claim was uncorroborated by any
separate competent evidence other than his testimony which was even doubtful. Pajarillos
apprehension that Evangeline will shoot him to stage a bank robbery has no basis at all. It
is therefore clear that the alleged threat of bank robbery was just a figment of Pajarillos
imagination which caused such unfounded unlawful aggression on his part.
As we have earlier held, Pajarillo failed to substantiate his claim that Evangeline was
seen roaming outside the vicinity of the bank and acting suspiciously prior to the shooting
incident. Evangelines death was merely due to Pajarillos negligence in shooting her on
his imagined threat that Evangeline will rob the bank.

https://lookaside.fbsbx.com/file/Book%20I_Consolidated%20C…hqsTJHSGnedWZ0OSOpxZmrtUVtu6RtpxVMMpL7Kt2IHuRsCvg 22/10/2018, 9W25 PM


Page 86 of 90
#77
G.R. No. 178115
July 28, 2014
PEOPLE OF THE PHILIPPINES
vs.
JOJO SUMILHIG, RICARDO SUMILHIG alias CARDING SUMILHIG, PASOT
SALOLI, ERIC ENOC, WARLITO MONTEBON,* and CIO LIMAMA

FACTS:
​One evening, Jerry Masaglang, Eugenio Santander, Eugenio’s son, Mario, were in
the living room of Eugenio’s house. They suddenly heard gunshots and saw six persons
firing at the kitchen. In the kitchen where members of the Santander family having
dinner. Jerry and Mario recognized the assailants to be the appellants and their co-
accused.
Before the accused ran off, Jojo shouted, "At last, I have retaliated!" The incident
caused the death of the children of Eugenio’s other son, Remegio Santander, 3-year old
Cresjoy, 8-year old Rolly. The teenagers, Marissa and Micel, sustained gunshot wounds.
Jojo denied the allegations and said he was in the house of his parents-in-law He
also said it was impossible for him to be at the scene of the crime since he could not walk
briskly due to a gunshot wound he earlier sustained in his left knee and anus. And that it
was only in January 1999 that he was able to walk without the aid of crutches. Jojo,
however, admitted harboring ill-will against the Santander clan for believing that they
were responsible for the massacre of his family in February 1998. Carding and Pasot both
claimed total ignorance of the incident.
The accused were charged with double murder and double frustrated murder.
​The Regional Trial Court found the accused guilty of the complex crime of double
murder and double frustrated murder. The Court of Appeals affirmed the trial court’s
decision, but changed it from a complex crime to only two counts of murder and two
counts of frustrated murder.

ISSUE:
1. Whether or not there was conspiracy and treachery.

HELD:
Conspiracy exists when two or more persons come to an agreement concerning
the commission of a felony and decide to commit it. It is not necessary to adduce
evidence of a previous agreement to commit a crime. Conspiracy may be shown through
circumstantial evidence, deduced from the mode and manner in which the offense was
perpetrated, or inferred from the acts of the accused themselves when such lead to a joint
purpose and design, concerted action, and community of interest. ​
Here, there is no proof of a previous agreement among the accused but there is a
series of events that clearly established conspiracy among them. First, they were all
armed with firearms. Second, they surreptitiously approached the crime scene. Third,
when they were within close range of the intended victims, they simultaneously
discharged their firearms. Fourth, they ceased firing at the same time and fled together.
Undoubtedly, their acts before, during and immediately after strafing the house of

https://lookaside.fbsbx.com/file/Book%20I_Consolidated%20C…hqsTJHSGnedWZ0OSOpxZmrtUVtu6RtpxVMMpL7Kt2IHuRsCvg 22/10/2018, 9W25 PM


Page 87 of 90
Eugenio evince their unanimity in design, intent and execution. Treachery attended the
commission of the crime.
There is treachery when the offender commits any of the crimes against the
person, employing means, methods or forms in the execution thereof which tend directly
and specially to insure the execution, without risk to himself arising from any defense
which the offended party might make."
Treachery is evident in this case as the suddenness and unexpectedness of the assault
deprived the victims of an opportunity to resist it or offer any defense of their persons.
This is considering that the victims were unaware that they would be attacked by
appellants with a hail of bullets from their firearms fired at close range. Indeed, "the
suddenness of the attack, without the slightest forewarning thereof, placed the victims in
such a position that they could not have defended themselves from the aggression.

https://lookaside.fbsbx.com/file/Book%20I_Consolidated%20C…hqsTJHSGnedWZ0OSOpxZmrtUVtu6RtpxVMMpL7Kt2IHuRsCvg 22/10/2018, 9W25 PM


Page 88 of 90
#78a
G.R. No. 108395
March 7, 1997
HEIRS OF THE LATE TEODORO GUARING, JR.
vs.
COURT OF APPEALS, PHILIPPINE RABBIT BUS LINES, INC., and ANGELES
CUEVAS

FACTS:
​There was a collision of three vehicles in the North Expressway in Pampanga. It
included a Mitsubishi Lancer carrying Teodoro Guaring, Jr. and Bonifacio Clemente,
Philippine Rabbit Bus No. 415, driven by Angeles Cuevas, and a Toyota Cressida,
carrying Sgt. Eligio Enriquez, Dolores Enriquez, Katherine Enriquez, Lilian Enriquex,
and Felix Candelaria.
​Guaring and Dolores died because of the accident.
​An action for damages was brought by the heirs of Teodoro Guaring, Jr. based on
quasi delict in the Regional Trial Court of Manila. The RTC favoured the heirs of
Guaring, Jr. and ordered the respondents to pay damages to the petitioners.
​The respondents appealed to the Court of Appeals. CA granted the appeal and set
aside RTC’s decision. It also dismissed the complaint against private respondents
Philippine Rabbit Bus Lines, Inc. and Cuevas. It based its decision on the decision
rendered by the Regional Trial Court at San Fernando, Pampanga, in the criminal case,
acquitting the bus driver Angeles Cuevas of reckless imprudence resulting in damage to
property and double homicide. CA held that since the basis of petitioners' action was the
alleged negligence of the bus driver, the latter's acquittal in the criminal case rendered the
civil case based on quasi delict untenable.
Hence, this petition.

ISSUE:
​Whether or not the criminal case extinguished the liability of Philippine Rabbit
Bus Line, Inc. and driver, Angeles Cuevas, for damages for the death of Teodoro
Guaring, Jr. based on reasonable doubt.

HELD:
No. The appellate court appears to have based its ruling on Rule 111, Section 2(b)
of the Rules of Criminal Procedure, which provides:
(b) Extinction of the penal action does not carry with it extinction of the civil,
unless the extinction proceeds from a declaration in a final judgment that the fact from
which the civil might arise did not exist.
This provision contemplates, however, a civil action arising from crime, whereas
the present action was instituted pursuant to Art. 2176 of the Civil Code, which provides:
Art. 2176. Whoever by act or omission causes damage to another, there being
fault or negligence, is obliged to pay for the damage done. Such fault or negligence, if
there is no pre-existing contractual relation between the parties, is called a quasi-delict
and is governed by the provisions of this Chapter.
It is now settled that acquittal of the accused, even if based on a finding that he is
not guilty, does not carry with it the extinction of the civil liability based on quasi delict.
The Supreme Court in a line of decisions has held that the civil case for damages was not
barred since the cause of action of the heirs was based on quasi delict. Even if damages
are sought on the basis of crime and not quasi delict, the acquittal of the bus driver will

https://lookaside.fbsbx.com/file/Book%20I_Consolidated%20C…hqsTJHSGnedWZ0OSOpxZmrtUVtu6RtpxVMMpL7Kt2IHuRsCvg 22/10/2018, 9W25 PM


Page 89 of 90
not bar recovery of damages because the acquittal was based not on a finding that he was
not guilty but only on reasonable doubt.
It has been held: “The judgment of acquittal extinguishes the liability of the
accused for damages only when it includes a declaration that the facts from which the
civil might arise did not exist. Thus, the civil liability is not extinguished by acquittal
where the acquittal is based on reasonable doubt as only preponderance of evidence is
required in civil cases; where the court expressly declares that the liability of the accused
is not criminal but only civil in nature as, for instance, in the felonies of estafa, theft, and
malicious mischief committed by certain relatives who thereby incur only civil liability;
and, where the civil liability does not arise from or is not based upon the criminal act of
which the accused was acquitted.

https://lookaside.fbsbx.com/file/Book%20I_Consolidated%20C…hqsTJHSGnedWZ0OSOpxZmrtUVtu6RtpxVMMpL7Kt2IHuRsCvg 22/10/2018, 9W25 PM


Page 90 of 90

Das könnte Ihnen auch gefallen